Lippincotts Illustrated Q&A Anatomy & Embryology MD2

¡Supera tus tareas y exámenes ahora con Quizwiz!

D: Cardiac nerve branches of the cervical sympathetic ganglia. This question asks which structure contains postsynaptic autonomic axons. Presynaptic sym- pathetic neurons that stimulate thoracic viscera synapse in the cervical and upper thoracic (T1 to T4) sympathetic trunk ganglia. Postsynaptic fibers leave the sympathetic trunk via the cardiac branches of the cervical ganglia and visceral branches of the T1 to T4 ganglia to enter the thoracic cavity. Choice A (Greater splanchnic nerves) is incorrect. These nerves branch from the T5 to T9 sympathetic trunk gan- glia. They carry presynaptic sympathetic fibers to the celiac and other upper abdominal plexuses. Choice B (Lumbar splanchnic nerves) is incorrect. These nerves branch from the L1 to L3 sympathetic chain ganglia. They convey presyn- aptic sympathetic fibers to preaortic ganglia in the abdomen and the hypogastric plexuses in the pelvis. Choice C (Pelvic splanchnic nerves) is incorrect. Pelvic splanchnic nerves are branches of the S2 to S4 ventral primary rami. They carry presynaptic parasympathetic fibers into the pelvic cavity for further distribution to the pelvis, perineum, and lower GI tract. Choice E (Cardiac nerve branches of the vagus nerve) is incorrect. The cardiac branches of the vagus nerves convey presynaptic parasympathetic axons into the thoracic cavity for distribution primarily to the heart.

22. Diabetes mellitus is a disease characterized by elevated blood glucose levels (hyperglycemia). Persistent hyperglycemia is associated with degeneration of Schwann cells, followed by axonal damage. In the autonomic nervous system, the struc- tures most susceptible to damage are the relatively smaller, nonmyelinated, postsynaptic axons. Which of the following structures are most likely to contain autonomic axons dam- aged secondary to diabetes mellitus? (A) Greater splanchnic nerves (B) Lumbar splanchnic nerves (C) Pelvic splanchnic nerves (D) Cardiac nerve branches of the cervical sympathetic ganglia (E) Cardiac nerve branches of the vagus nerve

E. Right ventricle. Due to the entry location of the ice pick, the right ventricle would be the most likely listed structure that is damaged. The right ventricle is the most ante- rior chamber of the heart, which makes it the most suscep- tible heart chamber during penetration wounds to the anterior chest. Based upon the wound site and the patient's symptoms, the fibrous pericardium, an unyielding sac that surrounds the heart, would also be damaged. Damage to the wall of the right ventricle would lead to a pericardial effusion (blood accumu- lation in the pericardial sac). Because the pericardial sac is fill- ing with blood and the fibrous pericardium is unyielding, the heart is being constricted and is unable to fill completely. This condition, known as cardiac tamponade, leads to the engorge- ment of veins of the face and neck because the pressure has limited the heart's intake. Fortunately, blood can be drained from the pericardial cavity via a pericardiocentesis, which will relieve the patient's symptoms temporarily until the heart wall is repaired surgically. If a pericardiocentesis does not occur, blood will continue to constrict the heart until it is unable to fill, and death will occur. Choice A (Right atrium) is incorrect. Due to the location of the wound to the left of the sternum at the 5th intercostal space, the right atrium, which forms the right margin of the heart, is not likely to be injured. Choice B (Left atrium) is incorrect. The left atrium is the most poste- rior heart chamber, so it is the least likely option to be dam- aged by this penetration wound on the anterior chest wall. Choice C (Aortic arch) is incorrect. The aortic arch is located above the 5th intercostal space, so it would not be damaged in this patient. Within the heart, the aorta is central in loca- tion, so even if the stab wound were at a higher level within the heart, the aorta could not be damaged without penetration through the right ventricle. Choice D (Left ventricle) is incor- rect. Due to the location of the wound at the left paraster- nal border of the 5th intercostal space, the left ventricle may seem to be a likely answer. However, the ventricular septum, which separates the left and right ventricles, runs obliquely in an anterolateral direction at approximately a 45-degree angle. The position of this septum, as well as the deviation of the heart to the left side, causes the right ventricle to be the most anterior chamber of the heart. Damage to the left ventricle, without involvement of the right ventricle, would only occur if the wound were located toward the apex of the heart, which runs laterally toward the midclavicular line.

36. A 23-year-old man was brought to the ER barely conscious but in great discomfort after being wounded by an ice pick in the anterior chest. His doctor records the entry wound as the fifth intercostal space, immediately left of the sternum. She also notes the veins of the face and neck are engorged with blood. The given chest X-ray showed massive enlargement of the patient's cardiac silhouette. Which of the following structures is most likely damaged due to the entry site of the ice pick? (A) Rightatrium (B) Leftatrium (C) Aorticarch (D) Leftventricle (E) Rightventricle

The answer is C: Tricuspid valve. The ideal placement of the stethoscope for auscultation of the tricuspid valve is at the 5th intercostal space slightly to the left of the sternal border, identified by the letter "C" in the photo. The tricuspid valve separates the right atrium and the right ventricle. Choice A (Aortic valve) is incorrect. The ideal placement of the stetho- scope for auscultation of the aortic valve is at the right second intercostal space slightly lateral to the sternal border, identi- fied by the letter "A" in the photo. The aortic valve is the only heart valve best heard by placing the stethoscope to the right of the midline of a patient. Choice B (Pulmonary valve) is incorrect. Auscultation of the pulmonary valve would be best heard by placing the stethoscope over the left 2nd intercostal space slightly to the left of the sternal border. This location is marked by the letter "B" in the photo. To listen to the tricuspid valve, the stethoscope needs to be placed more inferiorly at the fifth intercostal space slightly to the left of the sternal border. Choice D (Mitral valve) is incorrect. The ideal placement of the stethoscope for auscultation of the mitral (bicuspid) valve is at the left 5th intercostal space in the midclavicular line, which is located near the nipple. This location corresponds to the left ventricular area near the apex of the heart, iden- tified in the photo by the letter "D". Choice E (Left sternal border of heart) is incorrect. The location for auscultation of the left sternal border of the heart is identified by the letter "E" in the photo. To listen to the tricuspid valve, the stethoscope needs to be placed more inferiorly at the fifth intercostal space slightly to the left of the sternal border.

9. A 40-year-old man goes to his family physician for an annual examination. Which of the following locations is ideal for placement of the stethoscope for auscultation of the tricuspid valve of the heart?

The answer is D: Terminal sac period. The simple cuboidal epithelium within the terminal sacs differentiates into pneu- mocytes within the terminal sac period. The rapidly prolifer- ating capillary network makes intimate contact with the ter- minal sacs, and the blood-air barrier is established with Type I pneumocytes (or alveolar epithelial cells). These events take place in the terminal sac period, which runs from embryonic week 24 until birth and are crucial for the survival of an infant born prematurely. Administration of steroids to the infant in utero can promote lung maturation by increasing the prolif- eration of Type II pneumocytes (or alveolar epithelial cells), which secrete surfactant, a fluid capable of decreasing the sur- face tension at the air-alveolar interface. Due to the terminal sac period of lung maturation, a premature infant's survival rate will increase considerably from week 24 to 28 as the blood- air barrier continues to be established. Choice A (Embryonic period) is incorrect. At approximately 4 weeks of gestation, the respiratory diverticulum, or lung bud, begins to arise from the ventral wall of the foregut. Two longitudinal tracheoesopha- geal ridges fuse in the midline to form the tracheoesophageal septum, which separates the lung bud away from the foregut, specifically the esophagus, forming the trachea. The distal end of the trachea divides into two lateral bronchial buds. At the beginning of week 5, the bronchial buds elongate to form the primary (left and right main stem) bronchi, which further divide into the secondary bronchi. Choice B (Pseudoglandular period) is incorrect. The pseudoglandular period occurs from week 5 to 16 of gestation. During this period of lung maturation, the terminal bronchioles, which were established in the embryonic period of lung develop- ment, continue to divide to establish respiratory bronchioles and alveolar ducts. The vascular supply to the lungs also increases; however, respiration is not possible until there are enough capillaries within the lungs to enable sufficient gas exchange. Adequate gas exchange is not possible until the terminal sac period is reached at approximately the seventh month of gestation. Choice C (Canalicular period) is incor- rect. The canalicular period of lung maturation occurs from week 16 to 26 of gestation. During this period of lung devel- opment, the terminal sacs (primitive alveoli) form, and capil- laries within the lung tissue begins to establish close contact with the primitive alveoli. However, respiration is not possible until the terminal sac period when an adequate supply of cap- illaries exist to enable sufficient gas exchange. Adequate gas exchange is not possible until the seventh month of gestation. Choice E (Alveolar period) is incorrect. The alveolar period of lung maturation continues from month 8 of gestation and/ or birth until well into childhood. During this period of lung development, the number of mature alveoli within the lungs continues to increase in number and the blood-air barrier is now well established. However, it is during the terminal sac period of lung development (week 26-birth) when adequate gas exchange is established, making respiration possible in a premature infant.

A pregnant woman is in a car accident and goes into prema- ture labor. Her fetus is approximately 24 weeks in gestation. Her physicians administer her steroids and try to delay the birth of her baby. The survival rate of a premature baby rises significantly if the baby can reach 28 weeks gestation mainly due to the maturation of the lungs. What stage of lung matura- tion are the doctors hoping to reach, in which the blood-air barrier is beginning to be established? (A) Embryonicperiod (B) Pseudoglandularperiod (C) Canalicularperiod (D) Terminal sac period (E) Alveolar period

A. S1 spinal nerve roots. Approximately 90% of intervertebral disc herniations occur toward the bottom of the spine at the L4-5 or L5-S1 segments. This T1-weighted MRI reveals posterolateral herniation of the nucleus pulposus of the L5-S1 intervertebral disc, as indicated by the white arrow. Remember that a herniated nucleus pulposus may displace or disrupt the posterior longitudinal ligament and extend into the vertebral canal and impinge upon spinal nerve roots either within the vertebral canal or as they traverse the interverte- bral foramen (or both). When dealing with such cases in the lower lumbar regions, remember the formula "N+1." That is, N = the number of the intervertebral disc; +1 = the number of the spinal nerve roots primarily contacted by the herniation. Thus, the herniated L5-S1 intervertebral disc would impinge the S1 nerve roots, which resulted in the lower back pain, tingling numbness radiating down his right lower limb to the lateral part of his foot, and weakness when standing on the toes of his right foot. Choice B (S1 anterior primary ramus) is incorrect. The anterior (ventral) and posterior (dorsal) pri- mary rami are the first branches of the spinal nerves, located outside the vertebral canal. Regardless of the spinal segmen- tal level, these structures are not impinged by a herniation of the nucleus pulposus of the intervertebral disc. Choice C (L5 spinal nerve roots) is incorrect. Because of the acute angles that the lower lumbar nerve roots take in exiting the vertebral canal via the intervertebral foramina, the L5 spinal nerve roots would typically be held against the upper pedicle of L5, above the level of the L5-S1 disc. Thus, the L5 roots would not be impinged by a bulging L5-S1 disc. Therefore, impingement of the S1 nerve roots is a more likely scenario. Choice D (L5 posterior primary ramus) is incorrect. The ante- rior (ventral) and posterior (dorsal) primary rami are the first branches of the spinal nerves, located outside the vertebral canal. Regardless of the spinal segmental level, these structures are not impinged by a herniation of the nucleus pulposus of the intervertebral disc. Choice E (L5 anterior primary ramus) is incorrect. The anterior (ventral) and posterior (dorsal) pri- mary rami are the first branches of the spinal nerves, located outside the vertebral canal. Regardless of the spinal segmental level, these structures are not impinged by a herniation of the nucleus pulposus of the intervertebral disc.

After moving his parents' heavy furniture out of their house, a 38-year-old man experiences lower back pain and presents to his doctor with tingling numbness radiating down his right lower limb to the lateral part of his foot. A thorough physical examination reveals weakness when standing on the toes of his right foot. A T1-weighted MRI revealed an intervertebral disc herniation, as indicated by the white arrow. Which of the following structures is most likely impinged by this disc herniation? (A) S1 spinal nerve roots (B) S1 anterior primary ramus (C) L5 spinal nerve roots (D) L5 posterior primary ramus (E) L5 anterior primary ramus

E: Splenius capitis. The posterior (dorsal) pri- mary rami of spinal nerves innervate the embryonic epimere and all the skeletal muscles derived from it. The derivatives of the epimere constitute the deep (intrinsic) muscles of the back (or epaxial muscles), including the splenius capitis and cervicis muscles, suboccipital muscles, transversospinales muscles (including the semispinalis, multifidus, and rota- tores muscles), and several other small muscles. Thus, dam- age to the cervical posterior (dorsal) primary rami, specifically C2-6, would result in paralysis of the splenius capitis muscle. Choice A (Trapezius) is incorrect. The trapezius is a mem- ber of the superficial extrinsic layer of back muscles, which connect the upper limbs to the trunk. When these muscles contract, movements result in the upper limb. However, the trapezius muscle does not originate from either the epimere or hypomere, as evidenced by its innervation via the spi- nal accessory nerve (CN XI). Instead, it is likely related to postbranchial origins, along with the sternocleidomastoid. Thus, destruction of the cervical posterior (dorsal) primary rami will not directly affect the innervation of the trapezius muscle. Choice B (Latissimus dorsi) is incorrect. The latis- simus dorsi (L: widest muscle of back) is a member of the superficial extrinsic layer of back muscles, which connect the upper limbs to the trunk. When these muscles contract, movements result in the upper limb. The latissimus dorsi muscle is derived from hypomeres, which develop lateral and anterior to the vertebral axis. All of the muscles derived from hypomeres are innervated by the anterior (ventral) primary rami of spinal nerves. The latissimus dorsi muscle acts on the upper limb, and it migrates secondarily into the back. Due to being innervated by the anterior (ventral) primary rami, the latissimus dorsi muscle would not be affected by destruc- tion of the cervical posterior (dorsal) primary rami seen in this patient. Choice C (Levator scapulae) is incorrect. The levator scapulae muscle is a member of the superficial extrinsic layer of back muscles, which connect the upper limbs to the trunk. When these muscles contract, movements result in the upper limb. The levator scapulae muscle is derived from hypomeres, which develop lateral and anterior to the vertebral axis. All of the muscles derived from hypomeres are innervated by the anterior (ventral) primary rami of spinal nerves. Due to being innervated by the anterior (ventral) primary rami, the levator scapulae muscle would not be affected by destruction of the cervical posterior (dorsal) primary rami seen in this patient. Choice D (Rhomboid major) is incorrect. The rhomboid major muscle is a member of the superficial extrinsic layer of back muscles, which connect the upper limbs to the trunk. When these muscles contract, movements result in the upper limb. The rhomboid major muscle is derived from hypomeres, which develop lateral and anterior to the vertebral axis. All of the muscles derived from hypomeres are innervated by the anterior (ventral) primary rami of spinal nerves. Due to being innervated by the anterior (ventral) primary rami, the levator scapulae muscle would not be affected by destruction of the cervical posterior (dorsal) primary rami seen in this patient

As a result of multiple vertebral fractures incurred in an automobile crash, an 8-year-old girl suffers a series of torn posterior primary rami of spinal nerves C1-6. Which of the following muscles will be paralyzed as a result? (A) Trapezius (B) Latissimusdorsi (C) Levatorscapulae (D) Rhomboidmajor (E) Spleniuscapitus

B: Fracture of the dens axis (odontoid process). The sagittal CT clearly shows a Type II dens axis (odontoid process) fracture, which is the most common type of fracture (approximately 60% of cases) involving the dens. This fracture occurs at the base of the dens where it extends superiorly from the axis (or second cervical vertebra). The precise mechanism for odontoid fractures is unknown, but it most likely includes a combination of flexion, extension, and rotation. In this case, the patient's cervical spine was forced into a flexed position by the impact of the crash and then a hyperextended position with slight rotation due to the impact of her head on the steering wheel. The forces involved fractured the odontoid process at its base, but luckily, the fractured dens axis does not extend poste- rior to impinge upon the spinal cord. Patients with this type of odontoid fracture are placed in halo immobilization or undergo internal fixation (or odontoid screw fixation) to reattach the fractured axis dens to the body of the axis. Remember Steele's Rule of Thirds, which states one third of the atlas (C1) ring is occupied by the dens axis, one third by the spinal cord, and one third by the fluid-filled space and surrounding tissues of the cord. This extra space within the atlas ring explains why people with odontoid fractures may not have spinal cord injuries, but they will often feel unstable when they move their head sud- denly. The incidence of odontoid fractures approaches 15% of all cervical spine fractures. Choice A (Traumatic spondylolis- thesis of C2) is incorrect. Spondylolisthesis describes the ante- rior displacement of a vertebra in relation to the vertebra below it. Its meaning is derived from the Greek words "spondyl" (G: vertebrae) and "olisthesis" (G: slip). One of the most com- mon injuries of the cervical vertebrae involves the fracture of the vertebral arch of the axis (or 2nd cervical vertebra). The fracture usually occurs in the pars interarticularis or the bony column formed by the superior and inferior articular pro- cesses of the axis, and it is called a traumatic spondylolisthesis (or defect in the pars interarticularis) of C2 or hangman frac- ture. The precise mechanism for traumatic spondylolisthesis of C2 is the hyperextension of the head on the neck, not the combination of hyperflexion, rotation, and hyperextension of the head and neck seen in this MVA. It receives its alternative name, hangman fracture, because it is often seen in criminals executed by hanging due to the hyperextension of the head due to the placement of the noose under their chin. A sag- ittal CT would not be the best means to view a hangman fracture because the vertebral arch of the axis is not seen. A lateral cervical X-ray would be a better imaging technique to visualize a traumatic spondylolisthesis of C2 (or hangman fracture). Choice C (Atlanto-axial subluxation) is incorrect. Atlanto-axial subluxation, or the incomplete dislocation of the median atlanto-axial joint, occurs following the rupture of the transverse ligament of the atlas, which holds the dens axis (odontoid process) in place. Losing the integrity of the transverse ligament of the atlas can result in compression of the upper cervical spinal cord, leading to quadriplegia (paralysis of all four limbs) and even death (if the medulla of the brainstem is compressed). Atlanto-axial subluxation is prevalent in patients with Down syndrome (trisomy 21) but it can occur with other connective tissue disorders. In this patient, a fracture at the base of the odontoid process of the axis was noted on the sagittal CT scan. This fracture fragment is held in place by an intact transverse ligament of the atlas, so atlanto-axial subluxation did not occur in this patient. Choice D (Ruptured intervertebral disc between C1 and C2) is incorrect. A ruptured intervertebral disc between C1 and C2 is not possible because no disc exists between these two vertebrae. The given sagittal CT clearly shows a dens axis (odontoid process) fracture. The space between the fractured odontoid process and the body of the axis could be misinterpreted as a translucent space for an interverte- bral disc. Knowledge of the proper anatomy of the atlanto- axial joint would eliminate this option. Remember that a CT scan is valuable in assessing bone injuries, so a herniated disc would be better seen with an MRI. An MRI would be the best means to view damage to the intervertebral discs, the spinal cord and its nerve roots, and the ligaments of the spine. Choice E (No pathology is apparent on the CT scan) is incorrect. The given sagittal CT clearly shows a dens axis (odontoid process) fracture. The space between the fractured odontoid process and the body of the axis could be misin- terpreted as a translucent space for an intervertebral disc; however, no disc exists in the atlanto-axial joint. Knowledge of the proper anatomy of the atlanto-axial joint would enable easy elimination of this option.

During a head-on motor vehicular accident (MVA), the upper cervical vertebrae of a 34-year-old woman are flexed violently forward until her head impacts the steering wheel and is thrown into hyperextension and slight rotation. She is immo- bilized and brought to an ER. The given sagittal CT of her upper cervical vertebrae is viewed by a radiologist and reveals which of the following diagnoses? (A) Traumatic spondylolisthesis of C2 (B) Fracture of dens axis (odontoid process) (C) Atlanto-axialsubluxation (D) Ruptured intervertebral disc between C1 and C2 (E) No pathology is apparent on the CT scan

D: Meningomyelocele. A meningomyelocele is the most common form of spina bifida cystica. In this develop- mental disorder, the unfused portion of the vertebral column allows the meninges and the spinal cord to protrude through the structural defect, as noted in this longitudinal ultrasound scan. The protruded portion of the spinal cord is damaged (or not completed developed), and this defect may result in paral- ysis and loss of sensation below the level of the spinal cord defect. This condition is termed spina bifida cystica because of the characteristic presence of a cyst-like sac, composed of meninges and very thin skin, protruding from the defective area. The inclusion of a displaced portion of the spinal cord and nerve roots in the malformation designate this defect as a meningomyelocele. That is, the defect involves the meninges (meningo-), the spinal cord (myelo-), and the membranous sac (-cele). If it reaches full-term, this fetus will likely be paralyzed below the level of the meningomyelocele. In pre- natal tests, elevated levels of AFP in the amniotic fluid and maternal blood are strongly associated with the more severe forms of spina bifida, and an ultrasound scan is ordered to confirm the presence of a neural tube defect. Choice A (Meningocele) is incorrect. The least common form of spina bifida is a posterior meningocele (or meningeal cyst). In this developmental disorder, the bilateral neural arch elements fail to fuse completely in the dorsal midline, but the menin- ges protrude through the defect into a sac or cyst. Therefore, the multiple vertebral defects are accompanied with the presence of a cyst, which contains cerebrospinal fluid (CSF). In a meningocele, the spinal cord and nerve roots are typically in normal position, not protruding into the cyst, and there are usually no long-term effects on the individuals. In this fetus, the ultrasound scan showed a protruding cyst-like sac con- taining neural tissue, which would rule out a meningocele. Choice B (Anencephaly) is incorrect. Anencephaly is a severe neural tube defect in which the cephalic (head) end of the neu- ral tube fails to close in the embryo. This birth defect is lethal with the baby being born without a forebrain, calvaria, and scalp, leaving the remaining portions of the brain exposed. If proper prenatal care is available, most cases of anenceph- aly are detected by elevated maternal serum AFP levels and ultrasound scans during prenatal examinations. Elevated AFP levels are often correlated with neural tube defects, and the addition of folic acid to the diet of women in their childbearing years has been shown to reduce the incidence of neural tube defects. Because the given ultrasound scan localized the neural tube defect in the lower back, anencephaly can be eliminated as a valid answer. Choice C (Spina bifida occulta) is incor- rect. Spina bifida occulta is the mildest form of spina bifida (L: split spine). In this developmental disorder, the bilateral neural arch elements fail to fuse completely in the dorsal mid- line; however, the split in the vertebrae is so small that the meninges and elements of the spinal cord do not protrude through the defect. Because spina bifida occulta is such a mild defect, maternal serum AFP levels are usually not elevated during pregnancy. In fact, full-term babies with spina bifida occulta may be asymptomatic, and most individuals with this condition are unaware of having this mild developmental birth defect. Individuals with spina bifida occulta may possess a tuft of hair, dimple, lipoma, or even a birthmark in the overly- ing skin of the affected region. Some research suggests that approximately 10% of the general population have this mild- est form of spina bifida. Because maternal serum AFP levels would not be elevated in the case of spina bifida occulta, this defect can be eliminated as a valid answer. Choice E (Rachis- chisis) is incorrect. Rachischisis, or spina bifida with myelo- schisis, is the most severe form of spina bifida. In rachischi- sis, the protruded portions of the spinal cord are not afforded the enveloping protection of the meninges, so no cyst-like sac would be present. A physician would see a flattened mass of nervous tissue with no associated membranes, which makes the patient more prone to life-threatening infections. Because the presence of a cyst-like sac was noted in the ultrasound scan, rachischisis can be eliminated as a valid answer.

During a routine prenatal examination of a pregnant 16-year-old woman entering her second trimester (15 weeks of gestation), elevated maternal serum alpha-fetoprotein (AFP) levels were detected and later confirmed by amniocentesis. A follow-up diagnostic ultrasound examination reveals defects in the lower back of the fetus. In the given longitudinal ultrasound scan, open arrows indicate the bases of the vertebrae, and solid arrows point to a cyst-like sac containing protruding neural tissue. Which of the following terms best describes this neural tube defect? (A) Meningocele (B) Anencephaly (C) Spinabifidaocculta (D) Meningomyelocele (E) Rachischisis

A: Anterior longitudinal ligament. The anterior longitudinal ligament is a vertical connective tissue band that attaches along the anterior aspects of the vertebral bodies. Its peripheral fibers have strong attachments to the intervertebral discs. The anterior longitudinal ligament resists hyperexten- sion of the vertebral column. However, in this patient, the extreme forces involved with the hyperextension of the neck overpowers the resistance of this ligament, rupturing it as well as displacing part of the C4-5 intervertebral disc. In the given T2-weighted MRI, the anterior longitudinal ligament is repre- sented by a hypointense (dark band) signal located anterior to the vertebral column. However, the locations where the ante- rior longitudinal ligament is interrupted appear as an abnormal hyperintense (white) signal, which is evident anterior to the C5 vertebral body. Choice B (Posterior longitudinal ligament) is incorrect. The posterior longitudinal ligament runs verti- cally along the posterior aspect of the vertebral column, mirroring the position of the anterior longitudinal ligament located along the anterior aspect of the vertebral column. The posterior longitudinal ligament resists flexion of the vertebral column. Posterolateral herniation of the gelatinous nucleus pulposus through the anulus fibrosus of an intervertebral disc most often projects lateral to the strong attachment sites of the posterior longitudinal ligament. If the herniated disc compresses spinal nerve roots, then neck, back, and/or limb pain may be present. The T2-weighted MRI clearly shows dis- ruption of the anterior longitudinal ligament, evident by the abnormal hyperintense (white) signal located anterior to the C5 vertebral body. Choice C (Ligamentum flavum) is incor- rect. The ligamenta flava (L: yellow ligament) are paired liga- ments of yellow elastic fibrous tissue, which bind together the laminae of adjoining vertebrae and form the posterior wall of the vertebral canal. Because these ligaments resist flexion of the vertebral column, it is unlikely the ligamenta flava were damaged in this hyperextension injury of the neck. Choice D (Interspinous ligament) is incorrect. The inters- pinous ligament is composed of fibrous bands that connect the spinous processes of adjacent vertebrae. Because these ligaments resist flexion of the vertebral column, it is unlikely the interspinous ligaments were damaged in this hyperexten- sion injury of the neck. Choice E (Intertransverse ligament) is incorrect. The intertransverse ligament is one ligament that connects the transverse processes of adjacent vertebrae. Because the intertransverse ligaments resist contralateral bend- ing (abduction; lateral flexion) of the vertebrae, it is unlikely the intertransverse ligaments were damaged in this hyperex- tension injury of the neck.

Idling at a stoplight in his vintage car without headrests, a 71-year-old-man's car is struck from behind by a truck going approximately 30 mph (48 kph). The man is brought to the ER suffering from a severe hyperextension neck injury due to the crash. The given T2-weighted MRI shows a rupture of the anterior anulus of the C4-5 intervertebral disc, inflamma- tion of that disc (the white appearance), and a prevertebral hematoma, which compromised his airway and required intu- bation. Which of the following ligaments is disrupted in this injury? (A) Anterior longitudinal ligament (B) Posterior longitudinal ligament (C) Ligamentumflavum (D) Interspinousligament (E) Intertransverseligament

Right ventricle

What is the most anterior chamber of the heart?

11 The answer is C: Right subclavian artery. The right subclavian artery arises off the brachiocephalic trunk (a direct branch off of the arch of the aorta) and runs to the lateral border of the first rib. At this location, the subclavian artery becomes the axillary artery, which continues on to supply the upper limb. The right cervical rib (C7), depicted on the given X-ray, extends inferiorly in close relationship to the first rib. The sub- clavian artery typically passes over the cervical rib, causing the vessel to take a steeper route. When the shoulder is depressed (e.g., when carrying a heavy load), the subclavian artery may be compressed against the cervical rib, leading to the symp- toms present in the patient and resulting in thoracic outlet syndrome. Thoracic outlet syndrome causes pain, numbness, and paresthesia (tingling) in the neck and shoulder as well as the fingers of the hand. Moreover, the poor circulation in the right upper limb, due to the compression of the right subcla- vian artery, is causing the weakness (paresis) of the muscles of the hand. Choice A (Right common carotid artery) is incor- rect. The right common carotid artery arises off the brachio- cephalic trunk, which is a direct branch off of the arch of the aorta. The right common carotid artery ascends into the neck to split into the internal and external carotid arteries at the fourth cervical vertebra (C4). However, the common carotid is not closely associated with the first rib, so it will not be compromised by the presence of the right cervical rib (C7). Furthermore, compression of the right common carotid artery would lead to symptoms in the head and neck as well as affect the anterior circulation of the brain. However, the symptoms of this patient were localized to the right upper limb, so the right common carotid artery was not affected. Choice B (Bra- chiocephalic trunk) is incorrect. The brachiocephalic trunk, a direct branch off the arch of the aorta, gives rise to the right subclavian and common carotid arteries. If this artery were compressed by the right cervical rib (C7), the symptoms of the patient would include head and neck effects due to the involvement of the right common carotid artery. Choice D (Vertebral artery) is incorrect. The vertebral artery is the first branch off the subclavian artery, but this artery ascends through the cervical vertebral column to supply the posterior circulation of the brain. Compression of this artery would not explain the right upper limb symptoms, or the thoracic out- let syndrome, seen in this patient. Choice E (Right brachio- cephalic vein) is incorrect. Compression of the right brachio- cephalic vein would cause the superficial veins of the neck to become engorged due to the backflow of venous return to the heart. This sign is often seen in patients with congestive heart failure. However, compression of the right brachiocephalic vein would not explain the thoracic outlet syndrome seen in this patient because the right upper limb would still be receiv- ing its arterial supply. Therefore, blockage of this vein would not cause the symptoms seen in this patient.

11. A 56-year-old construction worker comes to his family physi- cian complaining of pain in the neck and shoulder, numbness and tingling in the right fingers, and a weak grip in his right hand. His symptoms are exacerbated by carrying heavy objects on his right shoulder. The given plain X-ray film indicates the presence of a right cervical (C7) rib in close approximation to the proximal part of the first rib. Given the signs and symp- toms of this patient, what structure is most likely impinged by the cervical rib? (A) Right common carotid artery (B) Brachiocephalictrunk (C) Right subclavian artery (D) Vertebralartery (E) Right brachiocephalic vein

The answer is D: Right coronary. The right coronary artery gives off arterial branches that usually supply both the SA and AV nodes. The SA node is usually supplied by the SA nodal artery, which branches off the proximal aspect of the right coronary artery. The AV nodal branch, which usually branches off the posterior interventricular branch of the right coronary artery, supplies the AV node. Iatrogenic injury to the right coronary artery would compromise the conduction system of the heart and lead to a heart block, as seen in this patient. Choice A (Anterior interventricular) is incorrect. The anterior interventricular artery, which is often called the left anterior descending (LAD) artery by clinicians, arises off the left coro- nary artery to supply the anterior aspects of both ventricles as it runs within the interventricular groove. In rare cases, its branches supply the AV bundle of the conduction system. Choice B (Left marginal) is incorrect. The left marginal artery branches off the circumflex branch of the left coronary artery to supply the left ventricle. This artery follows the left margin of the heart; however, iatrogenic injury to this artery would not cause the heart block seen in this patient. Choice C (Left coronary) is incorrect. The left coronary artery supplies the left atrium, most of the left ventricle, and some of the right ventricle. In rare cases, its branches supply the AV node of the conduction system, but the AV node is usually supplied by branches of the right coronary artery. Choice E (Circumflex branch) is incorrect. The circumflex branch of the left coro- nary artery follows the coronary sulcus around the left border of the heart to reach its posterior surface. This artery usually does not supply the conduction system of the heart, though it occasionally supplies blood to the SA node. However, the SA node is more commonly supplied by the sinuatrial node artery of the right coronary artery.

12. A 7-year-old boy with Down syndrome underwent surgery to repair a congenital ventricular septal defect. During this pro- cedure, iatrogenic injury to the coronary artery supplying the conduction system of the heart, including the sinuatrial (SA) and atrioventricular (AV) nodes, occurred, resulting in a heart block. What artery was most likely damaged by the surgeon? (A) Anteriorinterventricular (B) Leftmarginal (C) Leftcoronary (D) Rightcoronary (E) Circumflexbranch

A. Coarctation of the aorta. A coarctation of the aorta is a significant narrowing of the aorta distal to the origin of the left subclavian artery, as depicted on the given sagittal MRI. In coarctation of the aorta, blood flow into the descending aorta is diminished or blocked. However, arte- rial flow into the distal aorta is usually maintained via expan- sion of collateral circulation through the internal thoracic and intercostal arteries. The pressure dynamics in this patient, including hypertension in the upper limbs and diminished pressure in the lower limbs, represent the classic clinical signs of coarctation of the aorta. Many patients with coarctation of the aorta remain asymptomatic because of a persistent ductus arteriosus. When coarctation of the aorta is detected in new- borns, the child is often given prostaglandin agonists (PGE-1) to keep the ductus arteriosus patent until this defect can be fixed. Choice B (Tetralogy of Fallot) is incorrect. In tetralogy of Fallot, the right ventricular outflow experiences stenosis due to unequal division of the conus cordis and truncus arteriosus by the conotruncal septum. This anomaly presents with the following four characteristics: (1) overriding aorta, (2) valvular and infundibular pulmonary stenosis, (3) large VSD, and (4) hypertrophied right ventricle. However, this sagittal MRI depicts a narrowing of the aorta, which is not indicative of tetralogy of Fallot. Choice C (Ventricular septal defect [VSD]) is incorrect. A VSD is an abnormal opening in the septum between the right and left ventricles and would be seen by the cardiologist on the echocardiogram. However, the given MRI depicts a coarctation of the aorta, which is also consistent with the symptoms of the patient, including diminished pressure and inability to detect a pulse in the lower limbs. Choice D (Atrial septal defect [ASD]) is incorrect. An ASD is an abnormal opening in the septum between the right atrium and the left atrium, and this defect would be seen by the cardiologist on the echocardiogram. However, the given MRI depicts a coarc- tation of the aorta, which is also consistent with the symptoms of the patient, which are hypertension in the upper limbs and diminished pressure in the lower limbs. Choice E (Transpo- sition of the great vessels) is incorrect. Transposition of the aorta and pulmonary trunk is the result of failure of normal partitioning of the conus cordis and truncus arteriosus by the conotruncal septum. In this anomaly, the aorta originates from the right ventricle and the pulmonary trunk arises from the left ventricle; therefore, these major arteries are transposed from their normal positions. However, this sagittal MRI depicts a narrowing of the aorta distal to the left subclavian artery, or coarctation of the aorta.

13. During a newborn examination, a pediatrician is unable to detect a pulse in the groin or legs of an infant and notes the lower extremities are cold to the touch. The pulse and blood pressure of the upper limbs are significantly elevated from nor- mal. A pediatric cardiologist performs an echocardiogram and other imaging. The given sagittal MRI verifies the presence of what specific anomaly, as indicated by the white arrowhead? (A) Coarctation of the aorta (B) Tetralogy of Fallot (C) Ventricular septal defect (VSD) (D) Atrial septal defect (ASD) (E) Transposition of the great vessels

B: Membranous ventricular septal defect (VSD). The developing endocardial cushions constitute a tissue mass in the center of the heart that can be envisioned sche- matically as a cross-like formation. The crossbar (horizontal part) forms the atrioventricular canals and valves. The post (vertical part) forms portions of the atrial and ventricular septa. The lower limb of the vertical part forms the mem- branous part of the ventricular septum. Thus, defects in outgrowth of this component result in membranous VSDs. VSDs as a whole are the most common congenital cardiac malformations. Choice A (Atrial septal defect) is incor- rect. The upper limb of the vertical part of the endocardial cushion cross contributes to formation of the septum pri- mum in the common atrium. Extensions of the endocardial cushions here help to close the foramen (ostium) primum. Defects in this component result in primum type atrial sep- tal defects. Choice C (Transposition of the great vessels) is incorrect. This malformation is the result of failure of normal partitioning of the conus cordis and truncus arterio- sus. When the conotruncal septum runs directly downward through the conotruncal region instead of spiraling, the aorta originates from the right ventricle and the pulmonary trunk arises from the left ventricle. Thus, these major arter- ies are transposed from their normal positions. Choice D (Tricuspid atresia) is incorrect. In tricuspid atresia, the tri- cuspid valve is absent or fused and the right atrioventricu- lar orifice is obliterated. The horizontal part of the endo- cardial cushion cross forms the atrioventricular canals and valves. Thus, defective formation of the right limb of the endocardial cushion crossbar may produce tricuspid atresia. Choice E (Muscular ventricular septal defect) is incorrect. The medial walls of the two primitive ventricles merge to form the muscular part of the ventricular septum. This fuses with the lower limb of the endocardial cushions to form the complete ventricular septum. Muscular VSDs are more common than membranous VSDs, and most of these defects resolve during normal growth.

16. A genetic coding defect in an early embryo results in absence of the lower part of the vertical portion of the cross of the endocardial cushions in the developing heart. Which of the following malformations is most likely present? (A) Atrial septal defect (B) Membranous ventricular septal defect (VSD) (C) Transposition of the great vessels (D) Tricuspidstenosis (E) Muscular ventricular septal defect

D: It gives rise to the posterior interventricular (IV) artery. Coronary dominance refers to which coronary artery gives rise to the posterior IV artery, which is often termed the posterior descending artery by clinicians. When the right coronary artery gives rise to the posterior IV, the condition is right coronary dominance, which is the usual pattern in approximately 66% of the population. When the circumflex branch of the left coronary artery gives rise to the posterior IV, the condition is left coronary dominance, which occurs in approximately 15% of human hearts. Other patterns, including codominance, also occur. The posterior IV supplies neighboring aspects of both ventricles, including part of the ventricular septum. Thus, coronary dominance is important in determining the roles of each of the coronary arteries in supplying specific areas of the heart. Choice A (It gives rise to the anterior interventricular artery) is incorrect. The anterior IV and the circumflex branch are the terminal branches of the left coronary artery. The anterior IV supplies both ventricles and most of the ventricular septum. It usually anastomoses with the posterior IV across the inferior bor- der of the heart. Choice B (It is derived from the left 6th aortic arch) is incorrect. The coronary arteries are derived from both local epicardial tissue and migrating angioblasts. They are not related to the aortic arches. The left sixth aor- tic arch does give rise to the left pulmonary artery and the ductus arteriosus. Choice C (It supplies the right border of the heart) is incorrect. The right (acute) border of the heart is formed mainly by the right atrium, which is supplied by the right coronary artery. Coronary dominance is not related to supply to this area. Choice E (It supplies the margin of the heart) is incorrect. The margin (inferior border) of the heart is the nearly horizontal edge between the anterior (sternocostal) and inferior (diaphragmatic) surfaces of the heart. It is formed primarily by the right ventricle and is sup- plied largely by the marginal branch of the right coronary artery. Coronary dominance is not defined by the supply to this area.

24. During a surgery rotation, a 3rd year medical student is asked by a cardiovascular surgeon to explain what "left coronary dominance" means in relation to the left coronary artery. Which of the following explanations should be given by an astute student? (A) It gives rise to the anterior interventricular (IV) artery (B) It is derived from the left 6th aortic arch (C) It supplies the right border of the heart (D) It gives rise to the posterior interventricular artery (E) It supplies the margin of the heart

C: Traumatic spondylolysis of C2. Spondylolysis means a defect in the pars interarticularis of the affected ver- tebra (or C2 in this patient). The lateral cervical X-ray of this patient reveals a bilateral fracture of the axis (or C2) in the pars interarticularis or the bony column formed by the supe- rior and inferior articular processes of C2. This type of cervical fracture is often seen after hyperextension of the head in rela- tionship to the neck, and it is called a traumatic spondylolysis of C2 or hangman fracture. It receives its alternative name, hangman fracture, because it is often seen in criminals exe- cuted by hanging due to the hyperextension of the head due to the placement of the noose under their chin. This lateral cervi- cal film also reveals a traumatic spondylolisthesis of C2, which is sometimes evident following an acute fracture of the pars interarticularis of C2. Spondylolisthesis describes the anterior displacement of a vertebra in relation to the vertebra below it, and its meaning is derived from the Greek words "spondyl" (G: vertebrae) and "olisthesis" (G: slip). In the given X-ray, the vertebral body of C2 is displaced anterior in relationship to C3, which confirms the traumatic spondylolisthesis of C2. Therefore, the X-ray clearly shows a bilateral fracture of the pars interarticularis of C2 (or traumatic spondylolysis of C2 [hangman fracture]) and spondylolisthesis, anterior displace- ment, of C2 in relationship to the vertebral body of C3. This specific fracture was classified in 1981 by Effendi into a sub- type called "forward dislocation, axial arch fracture, Effendi Type II." Choice A (Ruptured anterior longitudinal ligament) is incorrect. The anterior longitudinal ligament is a vertically running band that attaches along the anterior sides of the ver- tebral bodies. Its peripheral fibers have strong attachments to the intervertebral discs. This ligament resists hyperextension of the vertebral column, and in this patient who experienced hyperextension of the head on the neck, the extreme forces involved with the impact of the clothes line on the patient's neck may have damaged the anterior longitudinal ligament, particularly due to the spondylolisthesis of C2. However, a ruptured anterior longitudinal ligament is not apparent on this lateral cervical X-ray, so the integrity of this ligament is unknown. The anterior longitudinal ligament would be bet- ter viewed with a T2-weighted MRI, which would show this ligament as a hypointense (dark black) band located anterior to the vertebral column. If damaged, the hypointense anterior longitudinal ligament will be interrupted by abnormal hyper- intense (white) signal. Choice B (Jefferson [burst] fracture of C1) is incorrect. The C1 vertebra, or atlas, is a closed ring with no vertebral body. Excessive vertical, or downward, force on the top of the head can fracture the anterior and posterior arches of C1 in multiple places, leading to a Jefferson (or burst) fracture of C1. Jefferson fractures of C1 often occur with axial loading force when the top of the head is impacted by a hard or heavy object. The downward force of the impact drives the lateral masses of the atlas lateral and fractures the anterior and posterior arches of C1. Because of the nature of this hyperex- tension injury and lack of visual evidence on the lateral cervi- cal film of damage to C1, a Jefferson (or burst) fracture of C1 is not the diagnosis. Choice D (Fracture of the dens axis [odon- toid process]) is incorrect. In the given lateral cervical X-ray, there is no apparent fracture of the dens (odontoid process) of the axis, or second cervical vertebra (C2). Because odontoid fractures are likely due to a combination of excessive flexion, extension, and some rotation within the cervical vertebrae, this patient, whose injury was caused by hyperextension of the head on the neck, is unlikely to have an odontoid fracture. The lateral cervical film confirms the presence of traumatic spondylolysis of C2, or a hangman fracture, and traumatic spondylolisthesis of C2. Choice E (Atlanto-axial subluxation) is incorrect. Atlanto-axial subluxation, or the incomplete dis- location of the median atlanto-axial joint, occurs following the rupture of the transverse ligament of the atlas, which holds the dens axis (odontoid process) in place. Losing the integrity of the transverse ligament of the atlas can result in compres- sion of the upper cervical spinal cord, leading to quadriplegia (paralysis of all four limbs) and even death (if the medulla of the brainstem is compressed). Atlanto-axial subluxation is prevalent in patients with Down syndrome (trisomy 21), but it can also occur with other connective tissue disorders. In this lateral cervical film, a bilateral fracture of the pars interarticu- laris of C2 is apparent, and the odontoid process of C2 is in its normal position, which implies an intact transverse ligament of the atlas. Therefore, atlanto-axial subluxation did not occur in this patient.

A helmet-less 24-year-old man was riding an all-terrain vehicle (ATV or quad bike) when a thin, unseen, horizontal clothes line impacted the man's neck under his chin, knocking him backward off the ATV. Due to his forward momentum at the time of impact, his head was hyperextended on his neck. Emergency medical technicians (EMTs) established his airway, stabilized his neck, and brought him to the emergency room. A lateral cervical X-ray revealed pathology within the cervical vertebrae. Based upon the nature of the accident and the evi- dence of the given X-ray, which of the following diagnoses is most probable? (A) Ruptured anterior longitudinal ligament (B) Jefferson (burst) fracture of C1 (C) Traumatic spondylolysis of C2 (D) Fracture of the dens axis (odontoid process) (E) Atlanto-axial subluxation

C. C6 spinal nerve roots. Classic posterolateral herniation of the nucleus pulposus of the intervertebral disc occurs most often within the C4-5 and L4-5 intervertebral discs. A herniated nucleus pulposus may displace or disrupt the posterior longitudinal ligament and extend into the verte- bral canal and impinge upon spinal nerve roots either within the vertebral canal or as they traverse the intervertebral fora- men (or both). When dealing with such cases in the cervical and lower lumbar regions, remember the formula "N+1." That is, N = the number of the intervertebral disc; +1 = the number of the spinal nerve roots primarily contacted by the herniation. Thus, the herniated C5-6 disc will most likely impinge upon the C6 spinal nerve roots. In the cervical region, this formula works because there are eight pairs of cervical spinal nerves but only seven cervical vertebrae. Each numbered spinal nerve exits above the matching numbered vertebra. Thus, the C6 cervical spinal nerve exits the intervertebral foramen formed by the C5 and C6 vertebrae, where it may be impinged. The formula also works in the low lumbar region because of the acute angles the nerve roots take in entering the intervertebral foramina. Here, the numbered spinal nerves exit below the matching vertebrae, so the L4 nerve roots enter the interverte- bral foramen between the L4 and L5 vertebrae. Because these lumbar nerve roots take a very acute turn to enter that open- ing, they are typically held against the upper pedicle and are superior to the level of the intervertebral disc. Thus, the L4 roots would not be impinged by a bulging L4-5 disc. There- fore, the L4-5 intervertebral disc would, instead, push primar- ily against the L5 roots as they align to enter the interverte- bral foramen formed by the L5 and S1 vertebrae. Choice A (C5 spinal nerve roots) is incorrect. Because of the "extra" pair of cervical spinal nerves (compared to the number of verte- brae), the cervical nerves exit the vertebral canal superior to their matching numbered vertebra. Thus, the C5 spinal nerve roots pass into the intervertebral foramen formed by the C4 and C5 vertebrae and would not be impinged by the bulging C5-6 intervertebral disc. Remember the N+1 rule. Choice B (C5 anterior primary ramus) is incorrect. The anterior (ven- tral) and posterior (dorsal) primary rami are the first branches of the spinal nerves, located outside the vertebral canal. Regardless of the spinal segmental level, these structures are not impinged by a herniation of the nucleus pulposus of the intervertebral disc. Choice D (C6 posterior primary ramus) is incorrect. The posterior (dorsal) and anterior (ventral) pri- mary rami are the first branches of the spinal nerves, located outside the vertebral canal. Regardless of the spinal segmen- tal level, these structures are not impinged by a herniation of the nucleus pulposus of the intervertebral disc. Choice E (C7 posterior horn segment of the spinal cord) is incorrect. The C7 posterior horn segment is inferior to the level of the herni- ated disc. Also, this structure is located on the opposite side of the spinal cord from the disc and would not be directly contacted by a herniation of the nucleus pulposus of the inter- vertebral disc.

After several months of engaging in a vigorous exercise pro- gram, a 28-year-old woman experiences periodic pain and muscle spasms in her left upper limb. Thorough physical and radiographic examinations by her primary care physi- cian and a consulting surgeon determine that she suffers a posterolateral herniation of the C5-6 intervertebral disc. Which of the following structures is this herniation most likely impinging? (A) C5 spinal nerve roots (B) C5 anterior primary ramus (C) C6 spinal nerve roots (D) C6 posterior primary ramus (E) C7 posterior horn segment of the spinal cord

The answer is D: Thoracic anterior roots. Horner syndrome presents with ptosis (drooping) of the upper eyelid, miosis (constricted or "pin-point" pupil), anhydrosis (warm, flushed, dry skin), and enophthalmosis (sunken eye), and the presence of this syndrome suggests damage to the pathway of sympa- thetic fibers to the head. The thoracic anterior roots are the only listed structures that carry sympathetic fibers destined for the head. The presynaptic sympathetic cell bodies reside in the intermediolateral (IML) cell column in the lateral horn of the gray matter within the T1-L2 (or L3) spinal segmen- tal levels. The sympathetic fibers destined for the head leave the spinal cord through thoracic anterior roots from T1 to T3. Choice A (Cervical anterior roots) is incorrect. The sympa- thetic nervous system is a thoracolumbar system, consisting of presynaptic cell bodies that reside in the IML cell column of spinal segmental levels of T1-L2 (or L3). The cervical ante- rior (ventral) roots is an incorrect answer because presynaptic sympathetic fibers ONLY reside in the anterior (ventral) roots of spinal nerves T1-L2 (or L3). Moreover, since the IML is somatotopically organized, the sympathetic fibers going to the head exist in the IML from T1 to T3 and ONLY run through the anterior (ventral) roots of these specific spinal nerves. The patient is exhibiting a loss of sympathetic innervation to the head, so damage to the cervical anterior roots would not result in Horner syndrome. Choice B (Thoracic posterior primary rami) is incorrect. Sympathetic fibers in the thoracic posterior (dorsal) primary rami have already synapsed in the sympa- thetic trunk and are not distributed to the head, so damage to these rami would not cause Horner syndrome. Postsynaptic sympathetic fibers pass through the thoracic posterior (dorsal) primary rami in order to vasoconstrict or vasodilate the vessels going to the areas supplied by these nerves, namely, the epaxial back muscles and the skin overlying these muscles. Choice C (Thoracic posterior roots) is incorrect. The autonomic ner- vous system is an entirely efferent (motor) system. The poste- rior (dorsal) roots contain only afferent (sensory) fibers, which eliminates these roots as a plausible choice. Choice E (Tho- racic gray rami communicantes) is incorrect. The thoracic gray rami communicantes convey postsynaptic sympathetic fibers (which have already synapsed in the sympathetic trunk) to the spinal nerves at the vertebral level at which they synapsed. These fibers distribute to the body wall and limbs. Therefore, the sympathetic fibers located in the thoracic gray rami com- municantes do not reach the head, and damage to these fibers would not cause Horner syndrome.

1. A 54-year-old man presents with loss of sympathetic innervation to the left side of his head, characteristic of Horner syndrome. An MRI revealed a left-sided pulmonary sulcus tumor with a location approximated by the black dot to which the arrow is pointing in the given photo. What structure is most likely compromised by the tumor? (A) Cervical anterior roots (B) Thoracic posterior primary rami (C) Thoracic posterior roots (D) Thoracic anterior roots (E) Thoracic gray rami communicantes

The answer is B: Transposition of the great vessels. The conus cordis and truncus arteriosus normally undergo an internal spiral partitioning (septation) that results in forma- tion of the ventricular outflow tracts, pulmonary trunk, and ascending aorta. Failure of the conotruncal septum to follow its normal spiral course results in transposition of the great vessels. In this case, the septum runs directly downward through the conotruncal region instead of spiraling, caus- ing the aorta to originate from the right ventricle and the pulmonary trunk to arise from the left ventricle. Alternately, displacement of the developing spiral septum causes unequal division of the conotruncal region, resulting in a narrow right ventricular outflow plus an overriding aorta. This condition is different from transposition of the great vessels and forms the basis for tetralogy of Fallot. Choice A (Double aortic arch) is incorrect. A double aortic arch is the result of the persistent right dorsal aorta connecting with the seventh intersegmental artery and the left dorsal aorta. This arrangement produces a vascular ring that surrounds both the trachea and esopha- gus. Choice C (Patent foramen ovale) is incorrect. A patent foramen ovale is a persistent opening in the atrial septum, resulting from failure of fusion of the septum secundum with the remnant of the septum primum. This defect allows post- natal exchange of blood between the two atria. Choice D (Ventricular septal defect) is incorrect. VSDs are gaps in the ventricular septum resulting from failure of fusion of the mus- cular walls of the primitive ventricles and/or failure of growth of tissue from the endocardial cushions. In either case, blood may be exchanged between the ventricles. VSDs are com- monly closely associated with other defects that result from mistakes in partitioning of the conotruncal region. However, they are not directly derived from such defects. Choice E (Ectopia cordis) is incorrect. In this rare malformation, the heart lies on the surface of the chest, which results from failure of the lateral body wall folds to meet and fuse in the anterior midline of the thorax.

10.Which of the following congenital defects is the direct outcome of malformation of the spiral partitioning of the conus cordis and truncus arteriosus? (A) Double aortic arch (B) Transposition of the great vessels (C) Patent foramen ovale (D) Ventricular septal defect (VSD) (E) Ectopiacordis

E: Esophageal atresia. Esophageal atresia is a congenital disorder in which the esophagus ends in a blind-ending pouch and is not connected to the stomach. In the given chest film, the catheter is resting in the blind- ending esophagus. The absence of air in the stomach also indicates an esophageal atresia. These malformations are detected during the infant's first feeding due to choking, coughing, and even sneezing. Excessive salivation may also be noted due to inability to swallow this secretion. In prena- tal examinations, esophageal atresia is often associated with polyhydramnios in the third trimester. Also, an esopha- geal atresia can present with or without a tracheoesopha- geal fistula. The latter scenario is more probable due to the appearance of normal lungs on X-ray. Choice A (Pyloric stenosis) is incorrect. Due to the location of the catheter on the X-ray, which is located within the esophagus, a pylo- ric stenosis (narrowing of the pyloric end of the stomach) is not probable. Infants usually do not present with symp- toms of pyloric stenosis until approximately 2 to 3 weeks after birth when they feed normally, but follow each feeding with nonbilious, projectile vomiting and a ravenous desire to continue to feed. Pyloric stenosis is usually detected with abdominal ultrasound and is prevalent in first-born Cau- casian boys. Choice B (Tracheoesophageal fistula [TEF]) is incorrect. A TEF is an abnormal connection between the trachea and the esophagus due to incomplete development of the tracheoesophageal septum, which normally separates the developing trachea from the esophageal portion of the foregut. TEFs are often seen with esophageal atresia; how- ever, in these cases, the baby would usually aspirate fluids into the lungs. In the given X-ray, there is no evidence of fluid accumulation in the lungs (pneumonia) secondary to the esophageal atresia. Normal lungs rule out the possibility of a TEF. Choice C (Large bowel obstruction) is incorrect. Large bowel obstruction would not explain the location of the catheter, which is stuck in the esophagus. Also, babies with large bowel obstruction would be able to feed; how- ever, they would present with bilious, projectile vomiting a few hours after feeding. This patient exhibited feeding problems immediately, which rules out large bowel obstruc- tion. Choice D (Respiratory distress syndrome [RDS]) is incorrect. RDS is common in babies of diabetic mothers and premature infants (born before 34 weeks of gestation). This patient is not exhibiting symptoms of RDS, which would include rapid breathing and shortness of breath. RDS rarely occurs in full-term infants.

15. A preterm neonate chokes and coughs throughout her first feeding with her mother with the breast milk regurgitating from the infant's mouth and nose. Noticing the difficulty feeding, a pediatrician tries to pass a catheter into the baby's stomach and meets resistance. A chest film is taken of the baby while in an incubator. The given radiograph reveals the location of the radiopaque catheter, shows no air within the stomach of the infant, and verifies normal lungs. What is the most likely diagnosis based upon the results of the X-ray? (A) Pyloricstenosis (B) Tracheoesophagealfistula(TEF) (C) Large bowel obstruction (D) Respiratory distress syndrome (RDS) (E) Esophagealatresia

C: Left main bronchus. The trachea, bronchi, and lungs appear as radiolucent (black) structures because of their air content. Left and right are differentiated by remem- bering that the conventional view of CT scans is from below, as if standing at the foot of the patient's bed and looking to the supine person's head. Also, the left main (primary) bronchus is more horizontally aligned, whereas the right main bronchus is more vertically aligned. Thus, the left main bronchus offers a more oblong profile in cross-sectional (axial) scans, while the right main bronchus is more circu- lar. Choice A is incorrect. The ascending aorta is identified. Note the position of this structure relative to the left ventri- cle. Choice B is incorrect. The right main bronchus is iden- tified. Note the differentiating features outlined previously. Choice D is incorrect. The azygos vein is identified. Note the size and position of the azygos vein slightly offset to the right anterior aspect of the vertebral body. Choice E is incorrect. The descending aorta is identified. Note the size and position of the aorta slightly offset to the left anterior aspect of the vertebral body.

17. Which of the labeled structures in the given CT scan of the thorax indicates the left main bronchus?

B: Left atrium. The left atrium forms most of the base (posterior aspect) of the heart. The esophagus passes immediately posterior to the left atrium, forming its major posterior relation. This relationship is readily seen in radiographs, especially following a barium swallow. Mitral valve stenosis restricts blood flow out of the left atrium, leading to dilation of this chamber. This dilation of the left atrium can compress the esophagus and impede swallow- ing. Heart valve damage is one possible outcome of rheu- matic fever and occurs more often in young girls. The given figure shows a barium swallow of a normal esophagus (right) demonstrating narrowing of the lumen at the sites of constriction, including the aortic arch, left main stem bron- chus, left atrium, and the diaphragmatic hiatus. Choice A (Right atrium) is incorrect. The right atrium contributes to a small portion of the base of the heart. However, it is not related to the esophagus. The right atrium forms the right border and right pulmonary surface of the heart. Choice C (Arch of the aorta) is incorrect. The esophagus passes posterior and to the right of the arch of the aorta. The aortic arch makes one of three normal thoracic impressions (con- strictions) on the thoracic part of the esophagus and may compress the esophagus. This close relationship is read- ily observed in PA chest radiographs following a barium swallow. However, this patient's mitral valve stenosis makes compression by the left atrium a more likely condition. Choice D (Pulmonary trunk) is incorrect. The pulmonary trunk arises from the right ventricle, on the anterior (ster- nocostal) surface of the heart. This vessel does not have a relation to the esophagus. Choice E (Superior vena cava) is incorrect. The superior vena cava lies immediately above the right border of the heart and empties into the right atrium. This large vessel does not have a relation to the esophagus.

18. A 14-year-old girl is having difficulty swallowing. Her case history reveals she has mitral valve stenosis related to rheu- matic fever. Which of the following structures is most likely compressing the esophagus? (A) Rightatrium (B) Leftatrium (C) Arch of the aorta (D) Pulmonarytrunk (E) Superior vena cava

D: Septum secundum. The septum secundum is the second fold of tissue that forms across the common atrium. However, it never forms a complete wall between the right and left atria. The opening left by the incomplete septum secundum, through which blood flows from the right atrium to the left atrium, is the foramen ovale. Ultimately, the septum secundum fuses with the remnant of the septum primum (the valve of the foramen ovale) to close the foramen ovale and form a complete atrial septum. Thus, a patent foramen ovale is the result of failure of fusion (to greater or lesser degrees) of the septum secundum with the remnant of the septum primum. Choice A (Endocardial cushions) is incorrect. Four endocardial cushions form and fuse together in the atrioventricular and conotruncal regions. They contrib- ute to the formation of the atrial and ventricular septa, the atrioventricular canals and valves, and the ventricular outflow tracts. Extensions of the endocardial cushions do contribute somewhat to the septum primum. However, the cushions have marginal contributions to the septum secundum. Choice B (Valve of the coronary sinus) is incorrect. This structure is not related to the atrial septum. The coronary sinus is derived from the left horn of the sinus venosus. The tissue flap that forms the valve of the coronary sinus is a derivative of the right sinus horn. Choice C (Ostium primum) is incorrect. The ostium primum is the opening between the septum primum and the endocardial cushions. It closes completely and does not contribute to the foramen ovale. Choice E (Septum spu- rium) is incorrect. This structure is not related to the foramen ovale. The septum spurium is a tissue ridge that forms at the junction of the right horn of the sinus venosus with the primi- tive atrium.

19. A newborn baby is diagnosed with tricuspid atresia. The given echocardiographic apical four-chamber view shows the abnormal valve plus its typical associated defects: a widely patent foramen ovale (double arrows), ventricular septal defect (single arrow), hypoplastic right ventricle, and hyper- trophied left ventricle. The patent foramen ovale most likely reflects a developmental failure of which of the following structures? (A) Endocardial cushions (B) Valve of the coronary sinus (C) Ostiumprimum (D) Septumsecundum (E) Septumspurium

B: Lymph nodes. Enlarged lymph nodes are the most likely cause of the widened mediastinum in this patient due to his inhalation of anthrax, which presents with the described flu-like symptoms. A widened mediastinum is seen in bacterial infections (as in this case), lymphoma, a soft tissue mass, and aortic aneurysm usually associated with trauma. Choice A (Aorta aneurysm rupture) is incor- rect. The patient came to the ER due to flu-like symptoms resulting from inhalation of anthrax, so a rupture of an aortic aneurysm is not likely because there was no reported trauma. During traumatic blows to the chest, the thoracic aorta can be ruptured, usually in the vicinity of the ligamentum arte- riosum. A widened mediastinum would be seen on X-ray in a patient experiencing this type of trauma due to the blood collecting in the mediastinum. Choice C (Thymus) is incor- rect. The thymus, a primary lymphatic organ, is located in the anterior mediastinum, and it reaches its height of growth in children approximately 8 years of age due to the matura- tion of the lymphatic system. After approximately age 12, the thymus regresses in size through a process known as accre- tion, in which the lymphatic tissue is slowly replaced with adipose fat. The thymus would not cause the abnormally wide space between the lungs seen in this patient because it rests in the anterior mediastinum. Choice D (Respiratory bronchioles) is incorrect. Respiratory bronchioles are located within the lungs and are continuous with alveolar ducts. Respiratory bronchioles can be identified by the presence of alveoli, which interrupt the epithelium. However, the lungs are located within the pulmonary cavity, so enlargement of the respiratory bronchioles would not cause a widened mediastinum. Choice E (Heart) is incorrect. An enlarged heart is often seen in patients with a history of heart disease, and this condition would cause an abnormally wide space between the lungs. However, there is no history of heart dis- ease. Thus, the mediastinum widening in this patient is most likely due to the inhalation anthrax diagnosis, which causes enlarged lymph nodes.

20. A 46-year-old postal worker was exposed unknowingly to the powdered form of bacteria Bacillus anthracis (anthrax). He comes to the ER with flu-like symptoms, including fever, chills, fatigue, headache, chest pain, and shortness of breath. A chest X-ray shows an abnormally wide space between the lungs. Enlargement of which of the following structures is the most likely cause of the widened mediastinum in this patient? (A) Aortic aneurysm rupture (B) Lymph nodes (C) Thymus (D) Respiratorybronchioles (E) Heart

B: Secretion of epinephrine. The lesions in question will disrupt right-side sympathetic outflow below the T5 level. The greater and lesser splanchnic nerves arise from the T5 to T11 segments of the sympathetic trunk. These nerves descend into the abdomen and distribute to upper abdominal viscera via their postsynaptic connections. One specific target is the adrenal (suprarenal) medulla, where sym- pathetic stimuli produce secretion of epinephrine. Choice A (Ability to produce cardiac deceleration) is incorrect. Cardiac deceleration is modulated by parasympathetic input sup- plied by cardiac branches of the vagus nerves. The cardiac nerves typically originate in the neck and descend into the thorax. Thus, the sympathetic trunk lesions described would not directly influence cardiac deceleration. Choice C (Secre- tion of gastric juices) is incorrect. Secretion of gastric juices is controlled by parasympathetic input supplied by both the anterior and posterior vagus nerves in the abdomen. Sym- pathetic trunk lesions would not compromise this pathway. Choice D (Ability to produce bronchoconstriction) is incor- rect. Bronchoconstriction is another parasympathetic func- tion controlled by the vagus nerves. Again, sympathetic trunk lesions would not compromise this pathway. Choice E (Vaso- dilation of the coronary arteries) is incorrect. Vasodilation of the coronary arteries is a result of sympathetic stimulation. However, the sympathetic supply to the heart is derived from cardiac branches off the cervical sympathetic trunk and vis- ceral branches off the T1 to T4 segments of the sympathetic trunk. These nerves pass into the thorax and distribute to the heart via the cardiac plexus. Thus, the sympathetic fibers des- tined for the coronary artery originate above the lesions and would not be affected by this condition.

21. A 12-year-old boy suffers an enlarging neurogenic tumor in the right posterior mediastinum, as shown in the given lateral view radiograph. This growth causes multiple lesions of the right thoracic sympathetic trunk below the T5 chain ganglion. Which of the following functions is most likely compromised? (A) Ability to produce cardiac deceleration (B) Secretion of epinephrine (C) Secretion of gastric juices (D) Ability to produce bronchoconstriction (E) Vasodilation of the coronary arteries

D: Left ventricle. The left ventricle is the struc- ture labeled "X" in the question and "D" on the given axial CT scan. Note the position of this chamber relative to the apex of the heart, which is pointing to the left. The left ventricle forms the left border of the heart, and its distinguishing fea- ture is its thick walls, needed to pump blood into the systemic circulation. Choice A (Ventricular septum) is incorrect. The ventricular septum is labeled "A" on the given axial CT scan. Note the position of the septum between the right and left ventricles and its oblique anterolateral orientation to the left at approximately a 45-degree angle. Choice B (Right ventricle) is incorrect. The right ventricle is labeled "B" on the given axial CT scan. Note the position of this chamber on the anterior (sternocostal) aspect of the heart. It is the most anterior cham- ber of the heart, which makes it susceptible to damage follow- ing penetration wounds to the anterior chest. Choice C (Right atrium) is incorrect. The right atrium is labeled "C" on the given axial CT scan. Note its posterolateral position relative to the right ventricle. The right atrium forms the right border of the heart. Choice E (Left atrium) is incorrect. The left atrium is labeled "E" on the given axial CT scan. It is located postero- medial to the left ventricle and is the most posterior chamber of the heart.

23. The structure indicated with the "X" on the given axial CT scan is which of the following structures (A) Ventricularseptum (B) Rightventricle (C) Rightatrium (D) Left ventricle (E) Left atrium

C: Endocardial cushions. The ventricular septum is composed of two parts: (1) an upper, smaller, thin- ner (membranous) part and (2) a lower, larger, thicker (mus- cular) part. The position of this defect at the upper end of the ventricular septum indicates it is located in the membra- nous part. The membranous portion of the septum is formed from outgrowths of the endocardial cushions. The endocar- dial cushion tissue fuses with the upper end of the muscular ventricular septum to close the interventricular foramen and form the complete ventricular septum. Choice A (Septum secundum) is incorrect. The septum secundum is the partition that forms most of the final atrial septum. It does not extend beyond the endocardial cushions into the ventricle. Choice B (Bulbus cordis) is incorrect. The bulbus cordis gives rise to the primitive right ventricle and to the ventricular outflow tracts. Its walls contribute to the muscular part of the ventricular septum but not to the membranous part. Choice D (Trun- cus arteriosus) is incorrect. The truncus arteriosus is the cra- nial end of the primitive heart tube. This structure forms the ascending aorta and the pulmonary trunk via its internal spiral septation. Choice E (Sinus venosus) is incorrect. The sinus venosus forms the caudal end of the primitive heart tube. It gives rise to structures related to the right atrium, including the smooth part of that chamber, the roots of the venae cavae, and the coronary sinus.

25. One finding during autopsy of a newborn infant that died soon after birth is a significant ventricular septal defect at the upper end of the interventricular septum, as shown in the given photo. This malformation may have been related to maldevelopment of which of the following embryonic structures? (A) Septumsecundum (B) Bulbuscordis (C) Endocardialcushions (D) Truncusarteriosus (E) Sinusvenosus

A: Diaphragm flattens. The diaphragm is the primary muscle of respiration. It contracts during inspiration and relaxes during expiration. When the diaphragm contracts, its two domes descend and flatten. Descent of the diaphragm increases the vertical dimension (height) of the thoracic cav- ity, resulting in increased intrathoracic volume, decreased intrathoracic pressure, and increased intra-abdominal pres- sure. These essential actions occur during both quiet and forced respiration. Choice B (Intercostal muscles relax) is incorrect. The actions of the individual intercostal muscles have been debated without clear resolution for many years. It appears that the external intercostals are most active during forced inspiration, whereas the internal and inner- most intercostals are most active during forced expiration. However, the main role of all the intercostals appears to be that of maintaining the spacing and rigidity of the intercos- tal spaces during all degrees (quiet and forced) and phases (inspiration and expiration) of respiration so that the rib cage moves as a complete unit in a coordinated fashion. Thus, we may consider the proposal that the external intercostals are most active in maintaining the intercostal spaces during inspiration, and the internal and innermost intercostals are most active in maintaining these spaces during expiration. Choice C (Ribs lower) is incorrect. The ribs are elevated dur- ing inspiration, especially in forced inspiration. Thus, the rib cage expands in upward and outward directions, resulting in increased anteroposterior and transverse dimensions, which, in turn, contributes to the increased intrathoracic volume and decreased intrathoracic pressure characteristic of inspiration. Choice D (Abdominal wall muscles contract) is incorrect. The three abdominal oblique muscles (external oblique, internal oblique, transversus abdominis) that form the anterolateral abdominal wall relax during inspiration and contract during expiration. Contraction of the diaphragm compresses the abdo- men and increases intra-abdominal pressure. Simultaneous relaxation of the abdominal muscles assists in accommodating these changes. Subsequent contraction of these muscles dur- ing expiration increases intra-abdominal pressure and elevates the relaxed diaphragm. Choice E (Horizontal dimension of the rib cage decreases) is incorrect. The ribs are elevated dur- ing inspiration, causing expansion of the rib cage in upward and outward directions. This expansion results in increased horizontal (transverse) and anteroposterior dimensions of the thoracic cavity.

26. A young medical student finds herself at a moment of great relaxation during her pre-exam meditation. Which of the following events is characteristic of the inspiratory phase of normal, quiet respiration? (A) Diaphragm flattens (B) Intercostal muscles relax (C) Ribslower (D) Abdominal wall muscles contract (E) Horizontal dimension of the rib cage decreases

C: Septum primum and the fused endocardial cushions. Closure of the foramen primum (ostium pri- mum) occurs when the free edge of the septum primum fuses with the endocardial cushions. While foramen secun- dum defects constitute the majority of atrial septal defects, foramen primum defects (primum atrial septal defects) do occur and often result in more severe conditions. The ventricular septal defects, seen on the given echocardio- gram, are often associated with Down syndrome and fetal alcohol syndrome. Choice A (Septum secundum and the septum primum) is incorrect. Because the foramen pri- mum closes before the septum secundum appears, the septum secundum does not play any role in closure of the foramen primum. Choice B (Septum secundum and the fused endocardial cushions) is incorrect. During nor- mal development, the foramen primum closes via fusion of the septum primum with the endocardial cushions. This event occurs before the septum secundum appears. Thus, the septum secundum plays no role in closure of the fora- men primum. Choice D (Septum primum and the septum spurium) is incorrect. The septum spurium is a fold in the right atrium formed by the right venous valve. It plays no role in partitioning of the atria. Thus, it plays no role in closure of the foramen primum. Choice E (Left inferior and right superior truncus swellings) is incorrect. The aorti- copulmonary septum, which divides the truncus arteriosus into the ascending aorta and pulmonary trunk, is formed by these two truncus swellings. They play no role in atrial septation.

27. A male baby is born with Down syndrome (trisomy 21) and associated cardiac defects. The given apical four-chamber view echocardiogram shows a complete atrioventricular canal defect that includes a primum atrial septal defect and a poste- rior inlet ventricular septal defect. What embryonic structures normally fuse to close the foramen primum? (A) Septum secundum and the septum primum (B) Septum secundum and the fused endocardial cushions (C) Septum primum and the fused endocardial cushions (D) Septum primum and the septum spurium (E) Left inferior and right superior truncus swellings

D: Spinal ganglion of T10. In this patient, her- pes zoster, or shingles, is a painful skin rash affecting the der- matome distribution pattern of the left 10th thoracic (T10) nerve, as evidenced by the involvement of the umbilicus. Shingles is seen in patients who have had previous exposure to the varicella zoster virus, which causes chickenpox in chil- dren or young adults. After the initial exposure to chickenpox, this virus can reside latent in ganglia of an individual for years. If this individual becomes immunocompromised, the skin (or dermatomes supplied by the infected ganglia) can develop shingles, a painful skin rash, which blisters, breaks open, crusts over, and then disappears. In this patient, the herpetic lesions were found in the sensory distribution of the left T10 nerve, which means the virus resides in this nerve's sensory ganglion, or the spinal ganglion of T10. Remember that two types of ganglia exist: sensory (afferent) and autonomic. The sensory ganglia, which are most often affected by shingles, are equivalent to the spinal (dorsal root) ganglia of spinal nerves in that they house typical pseudounipolar cell bodies of affer- ent neurons and do not contain synapses. If a spinal ganglion were infected by the varicella zoster virus, shingles may pres- ent in its dermatome distribution patterns when the patient is immunocompromised. Choice A (Paravertebral ganglion) is incorrect. The paravertebral (sympathetic) ganglia are located within the sympathetic trunk. They receive efferent fibers from the presynaptic (preganglionic) sympathetic neurons originating in the intermediolateral (IML) cell column of the thoracic and upper lumbar segments (T1-L2). Due to their location, these sympathetic ganglia are called paravertebral ganglia. Before it causes shingles (or herpes zoster), the vari- cella zoster virus resides latent in sensory ganglia for many years. Because the paravertebral ganglia are autonomic gan- glia, it is unlikely they would be responsible for the skin rash in this patient. Choice B (Prevertebral ganglion) is incorrect. The prevertebral ganglia are sympathetic ganglia that receive their name as they are located anterior to the vertebral column, as distinguished from the ganglia of the sympathetic trunk (paravertebral ganglia). Prevertebral ganglia exist near major branches of the abdominal aorta in the abdominopelvic cavity. These ganglia send postsynaptic (postganglionic) sympathetic fibers to the abdominopelvic organs with periarterial plexuses. Before it causes shingles (or herpes zoster), the varicella zoster virus resides latent in sensory ganglia for many years. Because the prevertebral ganglia are autonomic ganglia, it is unlikely they would be responsible for the skin rash in this patient. Choice C (Lumbar ganglion) is incorrect. The lumbar ganglia are paravertebral ganglia located in the abdominopelvic part of the sympathetic trunk. Before it causes shingles (or herpes zoster), the varicella zoster virus resides latent in sensory gan- glia for many years. Because the lumbar ganglia are autonomic ganglia, it is unlikely they would be responsible for the skin rash in this patient. Choice E (Spinal ganglion of L2) is incor- rect. The spinal (posterior root) ganglion is a sensory ganglion, so it could be infected by the varicella zoster virus and present with a painful, skin rash along its dermatome distribution pat- tern. The sensory distribution of L2 would extend along the back, hip, and extend into the anterior and medial aspects of the thigh. Because this patient presents with a skin rash at the level of the umbilicus, the spinal ganglion of T10 would be the source of this painful skin rash.

A 77-year-old man presents with shingles on his anterolateral abdomen and umbilicus, as shown in the figure. Shingles (or herpes zoster) is caused by the varicella zoster virus, which resides latent in sensory ganglia in the body for many years. When a patient is immunocompromised, this virus can cause a painful skin rash called shingles, which usually presents unilaterally along the infected nerve's dermatome distribution. Given the location of the rash, what ganglion is most likely affected in this patient? (A) Paravertebralganglion (B) Prevertebralganglion (C) Lumbarganglion (D) Spinal ganglion of T10 (E) Spinal ganglion of L2

E: Alveoli. The lower respiratory tract (larynx, trachea, bronchi, lungs) develops from a single ventral out- growth (respiratory diverticulum; lung bud) off the floor of the foregut. Thus, the epithelial lining of these structures, including the alveoli, is entirely derived from endoderm. The surrounding cartilage, muscle, and connective tissues are derived from the neighboring splanchnic mesoderm. Choice A (Nasal epithelium) is incorrect. The nasal cavities are derived from the nasal placodes, which are thickenings of surface ectoderm. Thus, the nasal epithelium is of ectoder- mal origin rather than endodermal origin. Choice B (Intrinsic laryngeal muscles) is incorrect. Intrinsic laryngeal muscles are all skeletal muscles derived from the 4th and 6th pharyngeal arches and of mesodermal origin. Due to their pharyngeal arch origins, these muscles are all innervated by the vagus nerve. Choice C (Bronchial cartilages) is incorrect. The bronchial cartilages that support the respiratory tree are derived from splanchnic mesoderm. Choice D (Trachealis muscle) is incor- rect. The smooth trachealis muscle is derived from surround- ing splanchnic mesoderm.

28. Which of the following components of the respiratory tract would be directly affected by a developmental failure of nor- mal differentiation of the foregut endoderm? (A) Nasalepithelium (B) Intrinsic laryngeal muscles (C) Bronchialcartilages (D) Trachealismuscle (E) Alveoli

E. Middle lobe of the lung. The right lung is composed of three lobes: superior, middle, and inferior. The lobes are separated by two fissures: oblique and horizontal. The oblique fissure extends from the level of approximately T2 posterior to the sixth costal cartilage anterior. The hori- zontal fissure runs anterior from the oblique fissure along the fourth rib and costal cartilage. This stab wound is located in the anterior chest wall between the oblique and horizontal fissures. Therefore the middle lobe of the lung is most likely pierced. Because of the angulations of the ribs and the fissures, the damaged lobe will vary depending on the longitude of the wound (e.g., depending on an entry point in the midclavicu- lar, midaxillary, or scapular line). Choice A (Superior lobe of the lung) is incorrect. The superior lobe extends down to the horizontal fissure, at approximately the 4th rib and costal car- tilage. Thus, it is above the wound in question. However, if this wound were on the left side, it would pierce the superior lobe because the left lung has only superior and inferior lobes. Choice B (Lingula of the lung) is incorrect. The lingula is a feature of only the left lung. It is a thin, tongue-like projec- tion of the anteroinferior margin of the superior lobe of the lung and is formed by the cardiac notch. Choice C (Inferior lobe of the lung) is incorrect. In the midclavicular line, the inferior lobe reaches as high as approximately the 6th rib and costal cartilage. Thus, it is below this wound point. Choice D (Apex of the lung) is incorrect. The apex is the rounded supe- rior end of the lung. It extends into the root of the neck, above the level of the first rib.

29. A 25-year-old man is brought to the emergency room after suffering a deep stab wound directly through the right 5th intercostal space in the midclavicular line. Which of the fol- lowing structures is most likely pierced? (A) Superior lobe of the lung (B) Lingula of the lung (C) Inferior lobe of the lung (D) Apex of the lung (E) Middle lobe of the lung

The answer is A: Anterior two thirds of interventricular septum. The artery that anatomists entitle the anterior inter- ventricular branch of the left coronary artery is often called the LAD artery by physicians in the clinic. This nomenclature is important because this artery represents the most com- monly occluded coronary artery in myocardial infarctions (heart attacks). As its anatomical name implies, it supplies the anterior two thirds of the interventricular septum, so this portion of the heart would most likely be damaged following an ischemic event due to a 90% stenosis of the LAD. Choice B (Posterior one third of interventricular septum) is incorrect. The posterior one third of the interventricular (IV) septum is usually supplied by the posterior IV branch of the right coronary artery, so this part of the heart would most likely not be damaged in this patient. Choice C (Atrioventricular [AV] node) is incorrect. The right coronary artery gives off the AV nodal branch in most cases, so this part of the con- ducting system of the heart would not be damaged in this patient. Choice D (Diaphragmatic surface of left ventricle) is incorrect. The diaphragmatic surface of the left ventricle is usually supplied by the posterior IV branch of the right coronary artery, so this part of the heart would most likely not be damaged in this patient. Choice E (Right atrium) is incorrect. The right atrium is supplied by the right coronary artery, so this part of the heart would not be damaged due to the location of the blockage in the LAD.

3. The given coronary artery angiogram is from a 68-year-old man with recurrent angina. It reveals 90% stenosis of the left anterior descending (LAD) artery, indicated by the white arrow. Based upon this finding, which portion of the heart is most likely susceptible to ischemic damage? (A) Anterior two thirds of interventricular septum (B) Posterior one third of interventricular septum (C) Atrioventricular (AV) node (D) Diaphragmatic surface of left ventricle (E) Rightatrium

B. Right atrium. The structures forming the cardiovascular shadow (mediastinal silhouette) comprise one of the most fundamental radiographic images seen in clinical studies. In PA radiographs such as seen here, the cardiovascu- lar shadow presents right and left borders composed of major vessels and chambers of the heart. The right atrium forms the convex border that makes up most of the lower half of the right side of the silhouette. Choice A (Superior vena cava) is incorrect. The superior vena cava makes up the right border immediately above the right atrium. Choice C (Inferior vena cava) is incorrect. The inferior vena cava appears as a small concavity on the right border between the right atrium and the diaphragm. Choice D (Arch of the aorta) is incorrect. The arch of the aorta (aortic knuckle; aortic knob) appears as a distinct rounded prominence at the upper end of the left bor- der. Choice E (Left ventricle) is incorrect. The left ventricle forms the large, slightly convex, lower part of the left border.

30. Which of the labeled structures in the given PA radiograph of the chest indicates the right atrium?

A: Opening of the coronary sinus. The coronary sinus is a large vein that receives most of the venous drainage of the heart. It empties into the right atrium, between the opening of the inferior vena cava and the right atrioventricular opening. The coronary sinus possesses a ridge-like valve at its aperture. Choice B (Openings of the pulmonary veins) is incorrect. The four pulmonary veins, carrying oxygenated blood from the lungs, open into the left atrium. The left atrium is the most posterior of the four heart chambers. Choice C (Septomarginal trabecula) is incorrect. The septomarginal trabecula (modera- tor band) is a segment of trabeculae carneae located in the right ventricle. It runs from the ventricular septum to the base of the anterior papillary muscle (located on the anterior mar- ginal wall of the ventricle). It prevents ("moderates") overd- istension of the right ventricle. Furthermore, it conveys the right limb of the atrioventricular bundle (Purkinje fibers) from the ventricular septum to the anterior wall of the ventricle. Choice D (Openings of the coronary arteries) is incorrect. The two coronary arteries arise from the ascending aorta, in the sinuses of the aortic valve. The right coronary arises from the right aortic sinus, whereas the left coronary arises from the left sinus. Choice E (Trabeculae carneae) is incorrect. These structures are located in both the right and left ventricles. They are a network of elevated ridges of myocardium that present a trabecular appearing surface within the ventricles.

31. A radiologist is examining a series of contrast-enhanced CT scans of a patient's thorax in evaluating findings of a hypertro- phied right heart. Which of the following structures is located in the pathologically enlarged right atrium? (A) Opening of the coronary sinus (B) Openings of the pulmonary veins (C) Septomarginaltrabecula (D) Openings of the coronary arteries (E) Trabeculaecarneae

E: Increase heart rate. Stimulation of para- sympathetic neurons normally results in deceleration of the heart rate. However, by blocking parasympathetic stimulation, atropine negates normal parasympathetic function and allows the sympathetic system to exert its functional effects (such as increasing heart rate) without balanced regulation. Choice A (Paralyze the diaphragm) is incorrect. The diaphragm is com- posed of skeletal muscle. Autonomic neurons act on smooth and cardiac muscle and glands. Thus, the diaphragm is not affected by parasympathetic action or lack thereof. Choice B (Stimulate bronchoconstriction) is incorrect. Parasympathetic activity normally stimulates bronchoconstriction. However, blocking parasympathetic input allows unopposed sym- pathetic action to cause the bronchi to dilate beyond their normal caliber, resulting in bronchodilatation. Choice C (Increase sweat gland secretions) is incorrect. Sweat glands do not receive parasympathetic supply. Thus, they are not affected by blocking parasympathetic stimulation. Choice D (Paralyze the intercostal muscles) is incorrect. All the inter- costal muscles are composed of skeletal muscle. They are not affected by parasympathetic action or lack thereof.

32. Atropine is a drug that acts to block stimulation of the receptors targeted by postsynaptic parasympathetic neu- rons. When it takes effect, atropine acts to do which of the following? (A) Paralyze the diaphragm (B) Stimulatebronchoconstriction (C) Increase sweat gland secretions (D) Paralyze the intercostal muscles (E) Increase heart rate

A: Right 2nd intercostal space at the right parasternal line. This location is optimal for listening to the aortic valve with a stethoscope. Auscultatory areas are those locations where the sounds of each heart valve can be best dif- ferentiated from the others. Because the sound of each valve tends to be carried in the direction that blood flows through it, the optimal auscultation point is over the location where blood flows after passing through each valve, and removed from a bony covering. Thus, the auscultation points are not the same as the anatomical surface projections of each valve. Choice B (Left 2nd intercostal space at the parasternal line) is incorrect. This location is optimal for listening to the pulmonary valve with a stethoscope. Choice C (Left 4th intercostal space at the parasternal line) is incorrect. This location is optimal for listening to the tricuspid valve with a stethoscope. Choice D (Left 5th intercostal space at the midclavicular line) is incor- rect. This location is optimal for listening to the mitral (bicus- pid) valve with a stethoscope. Choice E (Subxiphoid point) is incorrect. This location is not an auscultation point for any of the heart valves; however, it is the location for auscultation of epigastric sounds.

33. A 24-year-old man who has been overweight and sedentary most of his life experiences shortness of breath and cyanosis on exertion when he attempts to start an exercise program for the first time. A thorough physical examination by his physi- cian reveals an aortic valve defect and an aortic murmur. This murmur is best detected with the stethoscope placed over which of the indicated sites?

A: Double aortic arch. A double aortic arch results when the right dorsal aorta persists with a connection between the seventh intersegmental artery and the left dorsal aorta. This arrangement produces a vascular ring that sur- rounds both the trachea and esophagus. The ring compresses these tubes and causes difficulty with breathing and swallow- ing. Choice B (Persistent right 2nd aortic arch) is incorrect. Both the right and left second aortic arches mostly disappear during development, with small remnants forming the sta- pedial arteries in the head. A persistent second arch (either right or left) would be well removed from the trachea and esophagus and would not cause the compression seen here. Choice C (Coarctation of the aorta) is incorrect. Coarctation (significant narrowing) of the aorta, in either preductal or postductal forms, occurs distal to the origin of the left subcla- vian artery. Thus, this malformation is well removed from the trachea and esophagus. Even if the aortic arch were distended due to the coarctation, this dilatation of the aorta would not cause notable compression of the trachea or esophagus. Choice D (Patent ductus arteriosus) is incorrect. The ductus arteriosus lies in a similar position as a coarctation of the aorta. Patency here is far enough from the trachea and esophagus to not cause compression of these structures. Choice E (Per- sistent left fourth aortic arch) is incorrect. The left 4th aortic arch normally persists and forms a large part of the arch of the aorta. Thus, this condition is not a congenital malformation and does not compress the trachea or esophagus.

34. A newborn infant has difficulty breathing and swallowing. Radiographic examination reveals constrictions of both the trachea and esophagus in the same plane. Which of the fol- lowing congenital malformations is the most likely cause of this condition? (A) Double aortic arch (B) Persistent right 2nd aortic arch (C) Coarctation of the aorta (D) Patent ductus arteriosus (E) Persistent left 4th aortic arch

D: Right main bronchus. The main (primary) bronchi are the first branches of the trachea. They supply the right and left lungs. Because of the position of the heart, the main bronchi have different orientations in their routes to the lungs. The right main bronchus is shorter, wider, and more vertical. Thus, aspirated foreign bodies are more likely to drop into and become lodged in this tube. The given chest film shows air trapped within the right lung during expiration due to the presence of the peanut within the right main bron- chus. The right inferior lobar bronchus, a secondary bronchus that supplies the inferior lobe of the lung, is another common site for foreign objects to reside following accidental inhala- tion. Choice A (Larynx) is incorrect. The larynx is the com- pound chamber at the upper end of the lower respiratory tree, proximal to the trachea. Aspirated items may become lodged in the rima glottidis (the space between the true vocal folds). In such cases, the obstruction can usually be dislodged by using the Valsalva maneuver (i.e., by forcefully compressing the abdomen to rapidly expel air from the lungs). However, in this patient's case, the aspirated peanut is located in the thorax. If it were in the larynx, radiographs would show the peanut located in the neck. Choice B (Carina of the trachea) is incorrect. The carina is a small ridge at the inferior end of the trachea that separates the openings of the right and left main bronchi. Its mucosal covering is highly sensitive, and contact by an aspirated object stimulates the cough reflex. However, objects do not normally lodge at the carina of the trachea. Instead, they usually fall into the right main bron- chus. Choice C (Left main bronchus) is incorrect. Because the left main bronchus must extend past the heart, it is lon- ger, more narrow, and more horizontal than the right main bronchus. Thus, aspirated objects usually fall into the right main bronchus rather the left. Choice E (Right upper lobar bronchus) is incorrect. The right main bronchus divides into three lobar (secondary) bronchi, one to each lobe of the lung. The right superior lobar bronchus turns superiorly from the end of the main bronchus to enter the superior lobe. Thus, it is very difficult for an aspirated object to enter this tube. Instead, objects that pass beyond the main bronchus usually fall into the inferior lobar bronchus, which takes a more verti- cal descent.

35. A 2-year-old boy comes to the ER due to accidental inhalation of a peanut. Which of the following sites is the most likely location for the aspirated peanut in the thorax? (A) Larynx (B) Carina of the trachea (C) Left main bronchus (D) Right main bronchus (E) Right upper lobar bronchus

C: Superior mediastinum. The superior mediastinum is the large interpleural space above the plane running from the sternal angle to the T4 intervertebral disc. The symptoms seen in this patient suggest obstructed venous return from the head and neck, upper limbs, and thoracic walls. Venous drainage from these regions flows through the brachiocephalic veins and the azygos vein to converge in the superior vena cava (SVC). The SVC enters the right atrium through the right aspect of the superior mediastinum. Thus, the tumor in the right lung is most likely located in the superior lobe of the lung and is compressing the SVC in the superior mediastinum. Choice A (Anterior mediastinum) is incorrect. The anterior mediastinum is the small interpleural space between the sternum and the pericardial sac. The right internal thoracic vein (which collects the small anterior inter- costal veins) lies along the parasternal line inside the thoracic wall and could be compressed by an anterior lung tumor. However, such a condition would obstruct only a small por- tion of the thoracic wall drainage and would not produce the neck and upper limb effects seen here. Choice B (Posterior mediastinum) is incorrect. The posterior mediastinum is the large interpleural space posterior to the pericardial sac. The azygos vein ascends through roughly the right aspect of the posterior mediastinum and then curls through the superior mediastinum to join the SVC. A right lung tumor could com- press the azygos vein and notably obstruct venous drainage from the thoracic walls. However, collateral connections with the anterior intercostal veins could help relieve such obstruction. Further, obstruction of the azygos vein would not influence the neck and upper limb drainage and would not account for the full condition in this patient. Choice D (Hilum of the lung) is incorrect. The hilum is the slight con- cavity on the medial aspect of the lung where the root of the lung is attached. The root of the lung is the pleura-in- vested bundle of structures that enter and leave the lung at the hilum. These structures include the bronchi, pulmonary and bronchial vessels, lymphatics, and nerves. Compression of the root of the lung at the hilum would obstruct pulmo- nary functions, but would not produce any of the conditions noted in this patient. Choice E (Vena caval foramen of the diaphragm) is incorrect. The inferior vena cava (IVC) passes through the venal caval foramen of the diaphragm and imme- diately enters the right atrium. Compression of the IVC at this, or any other, point would not produce the effects seen in this patient.

37. A 65-year-old man presents with a swollen neck, marked edema in both upper limbs, and engorged and prominent intercostal veins. Subsequent examination reveals a tumor in his right lung. The tumor is likely compressing structures in which of the following regions? (A) Anteriormediastinum (B) Posteriormediastinum (C) Superiormediastinum (D) Hilum of the lung (E) Vena caval foramen of the diaphragm

C: Sinus venosus. The right horn of the embry- onic sinus venosus gives rise to the smooth part of the right atrium (the sinus venarum). In addition, it forms the roots of the superior and inferior vena cavae. Most of the left sinus horn is obliterated during development. Its major remnant forms the coronary sinus. Choice A (Pulmonary trunk) is incorrect. The pulmonary trunk and the ascending aorta are formed via the internal spiral septation of the truncus arteriosus. Thus, the pulmonary trunk is a derivative of an embryonic component rather than a source of other structures. Choice B (Conus cordis) is incorrect. The conus cordis is a derivative of the bulbus cordis. The conus cordis undergoes a complex spiral septation in close coordination with the septation of the ventricle and the truncus arteriosus. Ultimately, the conus cordis forms the right and left ventricular outflow channels. Choice D (Truncus arteriosus) is incorrect. The truncus arte- riosus is the cranial third of the bulbus cordis. It undergoes a spiral septation and forms the proximal parts of the pulmonary trunk and the aorta. Choice E (Ascending aorta) is incorrect. As noted previously, the ascending aorta and the pulmonary trunk are formed via the internal spiral septation of the trun- cus arteriosus. Neither gives rise to the left or right atrium.

38. An infant is born with an abnormally thin wall in the smooth part of the right atrium. This condition may be related to underdevelopment of which of the following embryonic structures? (A) Pulmonarytrunk (B) Conuscordis (C) Sinusvenosus (D) Truncusarteriosus (E) Ascendingaorta

D: Phrenic nerve. The phrenic nerve supplies innervation to the diaphragm and the diaphragmatic parietal peritoneum closely associated with the superior surface of the liver. Irritation of the diaphragm, due to the large hepatic (liver) abscess seen on the MRI, would cause the phrenic nerve to carry visceral sensory (pain) sensation back to its level of origin. The phrenic nerve is derived from cervical spinal nerves C3 to C5, and visceral pain can be interpreted as cutaneous pain along the dermatomes of C3 to C5. There- fore, the patient would experience referred pain in the upper right shoulder and neck regions due to irritation of the dia- phragm and subsequent involvement of the phrenic nerve. Choice A (Greater splanchnic nerve) is incorrect. The greater splanchnic nerve carries presynaptic sympathetic nerve fibers derived from the T5 to T9 vertebral levels. Visceral sensory fibers, associated with the greater splanchnic nerve and trav- eling retrograde with these visceral motor fibers, would refer pain to the T5 to T9 dermatomes, which corresponds to the anterolateral chest wall. Therefore, this nerve would not carry visceral sensory fibers that would refer pain to the right shoul- der region. Choice B (Lesser splanchnic nerve) is incorrect. The lesser splanchnic nerve carries presynaptic sympathetic nerve fibers derived from the T10 to T11 vertebral levels. Vis- ceral afferent fibers traveling along with the lesser splanch- nic nerve would refer pain to the T10 to T11 dermatomes, which is located at the umbilicus, well below the right shoul- der region. Choice C (Least splanchnic nerve) is incorrect. The least splanchnic nerve carries presynaptic sympathetic nerve fibers derived from the T12 vertebral level. Visceral afferent fibers traveling along with the least splanchnic nerve would refer pain to the T12 dermatome, which is located well below the right shoulder region. Choice E (Vagus nerve) is incorrect. The vagus nerve conveys presynaptic parasympa- thetic nerve fibers to supply the gastrointestinal tract up to the splenic flexure of the colon. It also conveys visceral affer- ent fibers, but these fibers are mainly for unconscious sensa- tions associated with reflexes. Therefore, these fibers would not be responsible for pain radiating to the shoulder region of this patient.

39. A 58-year-old woman presents with acute abdominal pain, which radiates superior to the right shoulder region. The given CT reveals a large hepatic abscess. Which of the follow- ing nerves conveys the visceral sensory fibers involved with the pain specific to the right shoulder region? (A) Greater splanchnic nerve (B) Lesser splanchnic nerve (C) Least splanchnic nerve (D) Phrenicnerve (E) Vagusnerve

B: Closed ductus arteriosus. Significant narrowing of the aorta distal to the origin of the left subcla- vian artery is termed a coarctation of the aorta. If the constric- tion of the aorta is proximal to the ductus arteriosus, which connects the pulmonary trunk to the arch of the aorta in fetal circulation, the condition is termed a preductal coarctation. If the constriction is distal to the ductus arteriosus, the con- dition is denoted as a postductal coarctation. In postductal coarctation (more common than the preductal form), the duc- tus arteriosus is normally closed and forms the ligamentum arteriosum. In this condition, blood flow into the descending aorta is blocked. However, arterial flow into the distal aorta is usually maintained via expansion of collateral circulation through the internal thoracic and intercostal arteries. The pressure dynamics in this situation account for the classic clinical signs of hypertension in the upper limbs (especially the right side) and diminished pulse and pressure in the lower limbs. Choice A (Patent ductus arteriosus) is incorrect. A pat- ent ductus arteriosus is characteristic of preductal coarctation of the aorta. In this case, the collateral circulatory expansions are not required because blood flows into the distal aorta through the patent ductus arteriosus. However, blood oxy- genation levels are altered due to persistence of this shunt. Interestingly, patients with preductal coarctation often appear healthy (asymptomatic). However, closure of the ductus arteriosus can be life threatening. Choice C (Patent ductus venosus) is incorrect. The ductus venosus is a fetal shunt between the umbilical vein and the inferior vena cava that allows blood from the placenta to mostly bypass the liver. It obliterates after birth and forms the ligamentum venosum. Because blood does not flow through this channel after birth, neither form of coarctation of the aorta affects this structure. Choice D (Patent foramen ovale) is incorrect. Arterial flow to the distal aorta is maintained in both forms of coarctation of the aorta. The pressure dynamics in each case are not suffi- cient to force persistence of a patent foramen ovale. If patency occurs, it is due to other factors. Choice E (Stenotic aortic valve) is incorrect. Formation of the aortic and pulmonary (semilunar) valves is related to septation of the conotruncal region. The mechanics behind constriction of the aorta are not the same as those related to failure of conotruncal septa- tion. Thus, stenosis of the semilunar valves is not a feature of coarctation of the aorta.

40. A 4-year-old boy presents with hypertension in the upper extremities and a diminished femoral pulse and pressure. Radiologic imaging reveals a postductal coarctation of the aorta. Which of the following is the most characteristic feature of this condition? (A) Patent ductus arteriosus (B) Closed ductus arteriosus (C) Patent ductus venosus (D) Patent foramen ovale (E) Stenotic aortic valve

C: Right vagus passes posterior to the root of the lung. The root of the lung is the pleura-invested bundle of structures that enter and leave the lung at the hilum, which includes the bronchi, pulmonary and bronchial vessels, lym- phatics, and nerves. Both the right and left vagus nerves descend in the thorax posterior to the root of each lung. At this depth, they are well positioned to contribute to the pulmonary and esophageal plexuses, and to follow the esophagus into the abdomen. Choice A (Right vagus passes onto the anterior aspect of the esophagus to become the ante- rior vagus) is incorrect. Due to the developmental clockwise rotation of the stomach, the lower esophagus also rotates, carrying the right vagus onto its posterior aspect, where the nerve becomes the posterior vagus. In the same way, the left vagus shifts to the anterior aspect of the esophagus and becomes the anterior vagus. Choice B (Left vagus passes through the anterior mediastinum) is incorrect. Both the left and right vagus nerves descend in the thorax posterior to the pericardial sac, that is, in the posterior mediastinum. Choice D (Left vagus passes across the posterior side of the aortic arch) is incorrect. The left vagus nerve crosses the ante- rolateral aspect of the aortic arch in passing from the superior to posterior mediastinum. The right vagus has no relation- ship to the aortic arch. Choice E (Right vagus passes through the middle mediastinum) is incorrect. Both the left and right vagus nerves descend in the thorax posterior to the peri- cardial sac, that is, in the posterior mediastinum. However, both the phrenic nerves and the pericardiacophrenic vessels are held in the pericardial sac and pass through the middle mediastinum.

41. In preparing for his Board Certification in thoracic surgery, a sur- geon reviews the geographic relations of critical structures in the chest. Which of the following is correct regarding the relation- ships of the vagus nerves in their passage through the thorax? (A) Right vagus passes onto the anterior aspect of the esopha- gus to become the anterior vagus (B) Left vagus passes through the anterior mediastinum (C) Right vagus passes posterior to the root of the lung (D) Left vagus passes across the posterior side of the aortic arch (E) Right vagus passes through the middle mediastinum

C: Diaphragmatic hernia. The traumatic force on the abdomen is sufficiently great to result in a tear of the right hemidiaphragm, allowing passage of the intra- abdominal organs into the thorax. In this patient, the presence of bowel sounds within the left thorax is the cardinal sign of a diaphragmatic hernia. The herniation of abdominal organs into the thoracic cavity would also account for the radiopacity seen on the chest film, the absence of audible breath sounds on the left side due to compression of the left lung, and the mediastinal shift due to increase in intrathoracic pressure on the left side. Choice A (Tension pneumothorax) is incorrect. A tension pneumothorax is a medical emergency in which air accumulates in the pleural cavity with each respiration, usu- ally resulting in a collapsed lung. This damage to the pari- etal pleura causes an increase in intrathoracic pressure, which pushes the contents of the mediastinum (a mediastinal shift) to the contralateral side and puts additional pressure on the unaffected lung, which was noted in this patient. A patient with a pneumothorax would exhibit respiratory distress; how- ever, the diagnosis of a pneumothorax does not explain the radiopacity in the left lower lung or the bowel sounds heard by the physician within the left thoracic cavity. Choice B (Hemopneumothorax) is incorrect. A hemopneumothorax is an accumulation of blood within in the pleural cavity. The presence of blood in the pleural cavity could lead to respira- tory distress, radiopacity in the area of the blood accumula- tion, and a mediastinal shift. However, this diagnosis does not explain the presence of bowel sounds in the left thoracic cavity. Choice D (Emphysema) is incorrect. Emphysema is a lung disease characterized by continual enlargement of air spaces within the lungs due to the destruction of lung tissue. This disease can lead to difficulty in breathing, but it can be ruled out as a diagnosis in this patient due to the presence of trauma and a localized radiopacity on the chest film. Radio- logic evidence for emphysema would be diffuse and would appear as radiolucent areas throughout the lungs. Emphysema would also not present with bowel sounds in the left thorax. Choice E (Left lower lobe pneumonia) is incorrect. Left lower lobe pneumonia would also present with radiopacity in the left lower lung on a chest film and difficulty in breathing. How- ever, this diagnosis could not explain the bowel sounds in the left thorax or the sudden onset of the symptoms following the accident.

42. A 59-year-old woman is brought to the ER in respiratory dis- tress after her abdomen impacted the steering wheel in a motor vehicle accident. Auscultation of the left chest reveals the absence of audible breath sounds, but the presence of bowel sounds is noted. The given chest X-ray reveals radiopacity in her lower left hemithorax and a rightward mediastinal shift. Which of the following diagnoses would best explain the patient's presentation? (A) Tensionpneumothorax (B) Hemopneumothorax (C) Diaphragmatichernia (D) Emphysema (E) Left lower lobe pneumonia

C: Left recurrent laryngeal. The ductus arte- riosus is a fetal shunt that connects the root of the left pul- monary artery to the inferior side of the arch of the aorta. It normally closes soon after birth, leaving a fibrous remnant, the ligamentum arteriosum. The left recurrent laryngeal nerve is a branch of the left vagus nerve. The recurrent laryngeal origi- nates at the inferior edge of the arch of the aorta, just lateral to the ductus arteriosus, curves under the aortic arch behind the ductus, and ascends into the neck. Thus, the left recurrent laryngeal nerve has an intimate relationship to the ductus arte- riosus that must be cared for in surgery in this area. Lesion of the nerve has major consequences, as it ultimately supplies motor fibers to most of the left side intrinsic laryngeal muscles and sensory fibers to much of the left side of the larynx. Choice A (Left vagus) is incorrect. The left vagus nerve crosses the lateral aspect of the arch of the aorta, descends posterior to the root of the lung, and continues along the esophagus. Although it passes near the ductus arteriosus, the left vagus does not have as close a relation as its recurrent laryngeal branch. Choice B (Left phrenic) is incorrect. This nerve crosses the left surface of the arch of the aorta, descends anterior to the root of the lung in the wall of the pericardial sac, and continues to the diaphragm. It is not closely related to the ductus arteriosus. Choice D (Right recurrent laryngeal) is incorrect. The right recurrent laryngeal nerve takes a very different path from its left mate. It branches from the right vagus anterior to the right subclavian artery, curves under the right subclavian artery, and ascends into the neck between the trachea and esophagus. It is far removed from the ductus arteriosus. Choice E (Right phrenic) is incorrect. This nerve descends along the lateral (right) side of the right brachiocephalic vein, superior vena cava, and pericardial sac. It is far removed from the ductus arteriosus.

43. A 7-day-old newborn with a diagnosis of patent ductus arteriosus undergoes surgery to ligate the ductus arteriosus. During the repair, the surgeon takes special care to avoid injury to a closely related nerve. The surgeon is protecting which of the following nerves? (A) Leftvagus (B) Leftphrenic (C) Left recurrent laryngeal (D) Right recurrent laryngeal (E) Right phrenic

D: The vein passes deep to the thymus gland. The left brachiocephalic vein runs obliquely across the superior mediastinum from its origin posterior to the left sternoclavicular joint to its termination posterior to the right first costal cartilage. Due to its course from the left side to right side of the chest, it is approximately twice the length of the right brachiocephalic vein. The left brachio- cephalic vein passes deep (posterior) to the thymus gland in its course toward the superior vena cava. The thymus is a major lymphoid organ situated in the anterior root of the neck, anterior part of the superior mediastinum, and ante- rior mediastinum. It achieves maximum size during puberty, then undergoes involution, and is replaced by fatty connec- tive tissue. Choice A (The vein lies posterior to the arch of the aorta) is incorrect. In its path across the superior medi- astinum, the left brachiocephalic vein runs just above the arch of the aorta, anterior to the roots of the three major branches of the arch of the aorta (brachiocephalic artery, left common carotid artery, left subclavian artery). Choice B (The brachiocephalic artery lies superficial to the vein) is incorrect. As it approaches its termination at the superior vena cava, the left brachiocephalic vein runs anterior to the brachiocephalic artery. Choice C (The vein crosses the anterior aspect of the manubrium of the sternum) is incor- rect. The left brachiocephalic vein is contained within the superior mediastinum. It crosses the posterior aspect of the manubrium of the sternum in its course from the left to the right side. Choice E (The apex of the left lung lies super- ficial to the vein) is incorrect. The apex is the superior end of the lung. It extends into the root of the neck, above the level of the first rib. The apex is superior and deep to the left brachiocephalic vein.

44. A medical resident is preparing to insert a central venous line into the left brachiocephalic vein of her 10-year-old patient. Which of the following is a correct relation of the left brachio- cephalic vein? (A) The vein lies posterior to the arch of the aorta (B) The brachiocephalic artery lies superficial to the vein (C) The vein crosses the anterior aspect of the manubrium of the sternum (D) The vein passes deep to the thymus gland (E) The apex of the left lung lies superficial to the vein

B: Superior lobe of right lung. The horizon- tal fissure separates the superior and middle lobes of the right lung. This fissure is located in the right lung only, and its location corresponds to the position of the right 4th rib and costal cartilage on the anterior chest wall. Therefore, a lobar pneumonia located above the horizontal fissure would indicate involvement of the superior lobe of the right lung. Remember, the right lung is composed of three lobes: supe- rior, middle, and inferior. The lobes are separated by two fis- sures: oblique and horizontal. The oblique fissure separates the lower lobe from the superior and middle lobes of the right lung, and it extends from the level of approximately T2 poste- rior to the sixth costal cartilage anterior. The given chest radio- graph shows an upper right lobar pneumonia, and the inferior extent of the inflammation and fluid accumulation is demar- cated by the horizontal fissure of the right lung. Choice A (Superior lobe of left lung) is incorrect. The left lung does not contain a horizontal fissure, so a lobar pneumonia located above the horizontal fissure would have to reside in the right lung. Choice C (Middle lobe of right lung) is incorrect. The horizontal fissure separates the superior and middle lobes of the right lung. This fissure is located in the right lung only, and its location corresponds to the position of the right fourth rib and costal cartilage on the anterior chest wall. A lobar pneumonia located above the horizontal fissure would indicate involvement of the superior lobe of the right lung, so the middle lobe of the lung would not be involved due to its position below the horizontal fissure. Choice D (Inferior lobe of right lung) is incorrect. The oblique fissure separates the lower lobe from the superior and middle lobes of the right lung, and this fissure extends from the level of approximately T2 posterior to the sixth costal cartilage anterior. Because the horizontal fissure separates the superior and middle lobes of the right lung, a lobar pneumonia located above the horizon- tal fissure would indicate involvement of the superior lobe of the right lung. Choice E (Inferior lobe of left lung) is incor- rect. The left lung does not contain a horizontal fissure, so a lobar pneumonia located above the horizontal fissure would have to reside in the right lung.

45. A chest X-ray reveals lobar pneumonia located over the hori- zontal fissure of the lung. Which of the following lobes of the lung would be inflamed and full of fluid? (A) Superior lobe of left lung (B) Superior lobe of right lung (C) Middle lobe of right lung (D) Inferior lobe of right lung (E) Inferior lobe of left lung

The answer is C: Splanchnic mesoderm. Cardiac progenitor cells migrate from the epiblast into the cranial portion of the splanchnic layer of the lateral plate mesoderm. In the meso- derm, the progenitor cells ultimately form paired cardiac pri- mordia, that is, the endocardial tubes. Subsequent lateral and cephalocaudal folding of the embryo causes merging of the paired endocardial tubes plus caudal shifting of the primitive heart tube and pericardial cavity. Choice A (Paraxial meso- derm) is incorrect. The paraxial mesoderm segments into somites along the long axis of the body wall. Each somite ultimately differentiates into three portions: sclerotome, myo- tome, and dermatome. Choice B (Intermediate mesoderm) is incorrect. The intermediate mesoderm is the small por- tion of mesoderm that connects the paraxial and lateral plate mesodermal areas. It differentiates into urogenital organs. Choice D (Parietal mesoderm) is incorrect. The lateral plate mesoderm splits into parietal (somatic) and visceral (splanch- nic) parts, separated by the intraembryonic body cavity. The parietal layer becomes associated with the overlying ectoderm to form the lateral body wall folds. Choice E (Extraembryonic mesoderm) is incorrect. This cell population appears outside the embryonic body proper. It forms a loose connective tissue zone that eventually results in the chorionic cavity.

6. An early term embryo spontaneously aborts due to incomplete lateral body wall folding and failure of the primitive paired endocardial tubes to merge and form the primitive heart tube. The endocardial tubes are derived from which of the following? (A) Paraxialmesoderm (B) Intermediatemesoderm (C) Splanchnicmesoderm (D) Parietalmesoderm (E) Extraembryonicmesoderm

The answer is A: Sinus venosus. The given diagram repre- sents the primitive five-division heart in the 4th week of development. The caudal (venous) pole of the heart is formed by the sinus venosus (structure A). The sinus venosus receives venous blood through its two extensions, the right and left sinus horns. As the heart develops further, most of the left sinus horn is obliterated. Its major remnant forms the cor- onary sinus, which receives most of the cardiac venous return. The right sinus horn persists to a greater degree and forms the smooth part of the right atrium (sinus venarum) and the roots of the venae cavae. Choice B (Primitive atrium) is incorrect. The primitive common atrium undergoes septa- tion and merges with the remnants of the sinus venosus to form the definitive right and left atria and auricles. The primi- tive right atrium forms the right auricle. The primitive left atrium forms the smooth part of the left atrium and the left auricle. Choice C (Primitive ventricle) is incorrect. The ven- tricle and the bulbus cordis of the primitive heart tube interact with each other in a complex septation process that produces the definitive right and left ventricles. The primitive ventricle forms most of the left ventricle. Choice D (Bulbus cordis) is incorrect. The bulbus cordis undergoes a complex spiral sep- tation in close coordination with the septation of the ventricle and the truncus arteriosus. Ultimately, the bulbus forms most of the definitive right ventricle plus the right and left ventricu- lar outflow channels. Choice E (Truncus arteriosus) is incor- rect. The truncus arteriosus is the cranial third of the bulbus cordis. It undergoes a spiral septation and forms the proximal parts of the pulmonary trunk and the aorta.

7. A 22-year-old woman gives birth to an infant with an abnormally small coronary sinus that limits cardiac venous drainage. The coronary sinus is derived from which of the labeled structures in the given diagram of the primitive heart?

The answer is D: Vagus nerves. The vagus nerves provide parasympathetic input to the thoracic viscera and to the abdominal viscera as far as the left colic (splenic) flexure. Car- diac branches of the vagi arise in the neck and descend into the thorax to supply the heart. Multiple additional branches arise in the thorax and supply the other thoracic viscera. Thus, parasympathetic input to the thorax is widespread from a great extent of the vagus nerves. Choice A (Cervical paraver- tebral ganglia) is incorrect. The paravertebral ganglia are the sympathetic ganglia. The cervical ganglia give rise to cardiac branches that provide much of the sympathetic supply to the thoracic viscera. Choice B (Greater splanchnic nerves) is incor- rect. These nerves are branches of the thoracic sympathetic trunk ganglia. They convey sympathetic fibers to the upper abdomen. Choice C (Gray rami communicantes) is incorrect. Gray and white rami communicantes are short connections between individual spinal nerves and the sympathetic trunk. The gray rami carry mainly postsynaptic sympathetic fibers. The white rami convey mainly presynaptic fibers. Choice E (Intercostal nerves) is incorrect. There is no parasympathetic innervation in the body wall and limbs. Thus, the intercostal nerves (supplying the thoracic and abdominal walls) do not carry parasympathetic fibers.

8. Disruption of parasympathetic input to the thorax may be related to damage to which of the following structures? (A) Cervical paravertebral ganglia (B) Greater splanchnic nerves (C) Gray rami communicantes (D) Vagusnerves (E) Intercostalnerves

A: Scoliosis. Scoliosis (G: crookedness) is abnormal lateral and rotational curvature of the spine that may present with uneven hips, shoulders, and rib cage, a head that is not centered over the pelvis, the entire body leaning to one side, back pain, and/or fatigue. The given Anterior- Posterior (AP) X-ray shows an S-shaped curvature of the spine or vertebral column in this patient. In this X-ray, no apparent vertebral anomaly is apparent, so the diagnosis is most likely adolescent idiopathic scoliosis, which has an onset of 10 to 18 years of age and has no known cause. Most cases of adolescent idiopathic scoliosis (with curvatures of less than 20 degrees) require no treatment; however, if the curvature goes above 25 degrees, a back brace can be implemented to slow the pro- gression of scoliosis. Choice B (Lordosis) is incorrect. Lordosis (G: bending backward) is an anteriorly convex curvature of the vertebral column (spine). The cervical and lumbar vertebral curvatures are normally lordotic; however, excessive lordotic curvature (also called hollow back, swayback, and saddle- back) can be caused by tight lower back muscles, excessive abdominal fat, and pregnancy. Excessive lordosis can lead to lower back pain and can be treated with strengthening of the abdominal muscles and hamstrings. Choice C (Kyphosis) is incorrect. Kyphosis (G: hump-back) is an anteriorly concave curvature of the vertebral column (spine). The thoracic and sacral vertebral curvatures are normally kyphotic; however, deformities of the spine, due to degenerative arthritis, osteo- porosis with compression fractures of the vertebrae, trauma, and developmental problems, can lead to excessive kyphotic curvature (or hunchback). Excessive kyphosis can cause pain and breathing difficulties. The given lateral X-ray shows an example of abnormal kyphotic curvature of the lumbar spine due to benign compression fractures of the L1 and L3 verte- brae secondary to osteoporosis. This disease is characterized by compromised bone strength and decreased bone mass. The affected L1 and L3 vertebrae depict an anterior wedging deformity, which would cause the patient to appear shorter (lose height). Choice D (Osteoporosis) is incorrect. Osteopo- rosis is a disease characterized by compromised bone strength and decreased bone mass, which can lead to an increased rate of fracture in the vertebral column, the mid-forearm, and more frequently the proximal femur resulting in hip fractures. Women are four times more likely to receive the prognosis of osteoporosis, with approximately 25% of women between 65 and 85 years old being diagnosed with osteoporosis. Due to the age of this 14-year-old patient and the findings of the nurse, osteoporosis is not likely. Choice E (Osteoarthritis) is incorrect. Osteoarthritis (or degenerative arthritis) erodes the articular cartilage in primarily weight-bearing joints. Because this type of arthritis is found in older populations, this diag- nosis is not likely in this 14-year-old patient, especially due to the symptoms noted by the nurse.

A 12-year-old girl is examined by a school nurse who notices the girl's right scapula is more prominent than the left, her head is not centered directly over the pelvis, and her right hip is raised and more prominent. When the girl is asked to bend forward at the waist, the nurse observes asymmetry of the trunk. Which of the following diagnoses is most likely? (A) Scoliosis (B) Lordosis (C) Kyphosis (D) Osteoporosis (E) Osteoarthritis

D: Spondylolysis. Spondylolysis refers to a defect in the pars interarticularis of the affected vertebra (or L5 in this patient). Its meaning is derived from the Greek words "spondylos" (G: vertebra) and "lysis" (G: loosening). The given right posterior oblique X-ray reveals a fracture in the isthmus (or neck) of the pars interarticularis or the bony column formed by the superior and inferior articular pro- cesses of L5. This fracture has the appearance of a "headless Scottie Dog" in this image. To visualize the Scottie Dog, look at the intact L4 vertebra, and remember that the ear is formed by the superior articular process, the head is formed by the transverse process, its eye is the radiopaque pedicle, its foreleg is the inferior articular process, its body is represented by the lamina and spinous process, and the hind leg of the dog is the opposite inferior articular process. Because there is a fracture within the isthmus (or neck) of the pars interarticularis of L5, spondylolysis of L5 is the proper diagnosis. Choice A (Anky- losing spondylitis) is incorrect. Ankylosing spondylitis is arthritis of the spine, which presents with inflammation, stiff- ening of the joint, and potentially bony ankylosis (synostosis) due to ossification of the posterior and anterior longitudinal ligaments. Ankylosing spondylitis is derived from the Greek words "ankylosis" (G: stiffening of the joint), "spondylos" (G: vertebra), and "itis" (G: inflammation). Ankylosing spon- dylitis, a degenerative joint disease of the spine, with an autoimmune component. However, this diagnosis is not sup- ported by the given lumbar spine X-ray. Choice B (High-grade spondylolisthesis) is incorrect. Spondylolisthesis describes the anterior displacement of a vertebra in relation to the verte- bra below it. Its meaning is derived from the Greek words "spondylos" (G: vertebra) and "olisthesis" (G: slipping and falling). A high-grade (III or IV) spondylolisthesis implies the anteriorly displaced posterior edge of the L5 vertebral body is located anterior to the center of the S1 vertebral body. A high-grade spondylolisthesis means the vertebral body of L5 is displaced anteriorly by the width of more than half of the vertebral body of S1. The given lumbar spine X-ray does not support this diagnosis. Choice C (Spondyloptosis) is incor- rect. Spondyloptosis is the most extreme type of spondylolis- thesis in which the posterior edge of the L5 vertebra is dis- located anterior to the entire S1 vertebral body, so it would fall off the vertebral column. "Spondyloptosis" is derived from the Greek words "spondylos" (G: vertebra) and "ptosis" (G: falling). This extreme form of spondylolisthesis was not supported by the given lumbar spine X-ray. Choice E (Spon- dylosis) is incorrect. Spondylosis is a term that implies stiffen- ing of the spine (ankylosis) due to any lesion of the spine of a degenerative nature. Due to the young age of this male athlete, a degenerative joint disease of the spine described by the word "spondylosis" is highly unlikely. Furthermore, this diagnosis is not supported by the given lumbar spine X-ray.

A 16-year-old male soccer player was brought to the ER because of the acute lower back pain he experienced after performing a high-velocity kick of the ball. The given right posterior oblique X-ray reveals pathology within the L5 verte- bra that has the appearance of a "headless Scottie Dog" when outlined. Based upon the X-ray results, which of the following diagnoses is the most likely cause of his lower back pain? (A) Ankylosingspondylitis (B) High-gradespondylolisthesis (C) Spondyloptosis (D) Spondylolysis (E) Spondylosis

B: Trapezius, scapula, latissimus dorsi. The triangle of auscultation is a space in the back bounded by the lateral edge of the trapezius muscle, medial border of the scap- ula, and upper edge of the latissimus dorsi muscle. Placing a stethoscope within the triangle of auscultation enables the medical student to hear breathing sounds because (1) this site offers a gap between layers of bone and muscle, and (2) the location lies directly over the midposterior chest wall and the lung, which is ideal for auscultation. Choice A (Deltoid, latis- simus dorsi, scapula) is incorrect. A triangle marked by these boundaries is located over the posterior aspect of the axilla. A medical student would be unable to hear breathing sounds of the lungs at this location. Choice C (Latissimus dorsi, ilium, external abdominal oblique) is incorrect. The lumbar triangle is the space bounded by the lower edge of the latis- simus dorsi muscle, the iliac crest of the coxal (hip) bone, and the posterior border of the external abdominal oblique muscle. An abdominal hernia may protrude through the lum- bar triangle, a weak spot in the abdominal wall. The lumbar triangle also offers a surgical pathway through the back for access to the kidney. Choice D (Rhomboids, levator scapulae, splenius capitis) is incorrect. The intermuscular space marked by these muscles is located at the base of the neck. A medical student would be unable to hear breathing sounds of the lungs at this location. Choice E (Longissimus, rhomboids, vertebral spinous processes) is incorrect. These boundaries form a site just off the dorsal midline. Listening for breathing sounds of the lungs at this site is not optimal.

An internal medicine attending physician asks a medical student student to place the bell of her stethoscope on the triangle of auscultation to hear a patient's breathing sounds. Which of the following structures make up the boundaries of this triangle? (A) Deltoid, latissimus dorsi, scapula (B) Trapezius, scapula, latissimus dorsi (C) Latissimus dorsi, ilium, external abdominal oblique (D) Rhomboids, levator scapulae, splenius capitis (E) Longissimus, rhomboids, vertebral spinous processes

D: Latissimus dorsi. The latissimus dorsi (L: widest muscle of back) is a large, fan-shaped muscle and a member of the superficial extrinsic layer of back muscles, which connect the upper limbs to the trunk. The chin-up movement described includes lifting the body toward the upper limb, which is a powerful extension action. In con- junction with the pectoralis major muscle, the latissimus dorsi muscle raises the trunk to the arm, which is crucial in performing chin-ups or climbing a tree. Remember that the superficial layer of back muscles is composed of upper limb muscles that take large bases of origin in the back. Athletes who make heavy use of extension and medial rotation of the arm (as in chin-ups or climbing in conditional exercises) typi- cally have well-developed "lats" that give the classic "V" shape to the back. Choice A (Serratus posterior superior) is incor- rect. The serratus posterior superior muscle is a member of the intermediate layer of back muscles that lies deep to the rhom- boid muscles. Due to its insertion into the superior borders of the 2nd to 5th ribs, it is said to elevate the ribs to increase the diameter of the thorax, acting as an accessory muscle of respiration. Due to its small size and lack of attachment to the appendicular skeleton, it would not be the primary muscle involved with performing chin-ups. Choice B (Rhomboid major) is incorrect. The rhomboid major and minor muscles act to retract (adduct) the scapula. In the chin-up movement, the trunk of the body is lifting toward the upper limb, and the rhomboids are important in stabilization of the scapula to allow the latissimus dorsi muscle to raise the trunk toward the upper limb. However, these muscles are not the prime agonist in this climbing movement. Choice C (Levator scapulae) is incorrect. The levator scapulae muscle acts mainly to elevate the scapula. During the chin-up movement, the levator scap- ulae muscle also aids in downward rotation of the scapula, which would aid the latissimus dorsi muscle in raising the trunk toward the upper limb. However, the levator scapulae muscle is not the prime agonist in this climbing movement. Choice E (Longissimus) is incorrect. The longissimus muscle is one of the three components of the erector spinae group of deep back muscles. Its primary action is to extend the back. Because it does not attach or originate within the upper limb, the longissimus muscle is unable to lift the trunk toward the upper limb, which is what defines a chin-up.

A 17-year-old gymnast grips a high bar with his arms out- stretched and begins to pull himself straight upward to the level of the bar, as in doing a chin-up. Which of the following muscles is the prime agonist in this action? (A) Serratus posterior superior (B) Rhomboidmajor (C) Levatorscapulae (D) Latissimusdorsi (E) Longissimus

B: Folate. Supplementing a diet with folic acid (vitamin B9 or folacin) has been shown to reduce the incidence of NTDs up to 70%. Neural tube clo- sure occurs within the first 28 days after conception, and at this time in the pregnancy, many mothers do not realize they are pregnant. Women anticipating getting pregnant should take 400 mg of folic acid daily beginning 3 months prior to conception and continue the folic acid supplement throughout their pregnancy. NTD prevention is best achieved by adequate daily folic acid intake throughout the reproduc- tive years. In this neonate, the sagittal MRI revealed a neural tube defect called a meningomyelocele, so the defect in the cervical region involves the meninges (meningo-), the spinal cord (myelo-) and the membranous sac (-cele). A meningo- myelocele is the most common form of spina bifida cystica. Choice A (Valproic acid [Depakote]) is incorrect. Valproic acid (brand name Depakote) is an anticonvulsant and mood-stabi- lizing drug used in the treatment of bipolar disorder, depres- sion, and epilepsy. Exposure to Valproic acid, during the first month of pregnancy, has been shown to have a teratogenic effect on the fetus, including increasing the incidence of neu- ral tube defects. Choice C (Cephalexin [Keflex]) is incorrect. Cephalexin (brand name Keflex) is an antibiotic used to fight bacterial infections, including upper respiratory, ear, skin, and urinary tract infections. Exposure to Cephalexin, during the first month of pregnancy, has been shown to have a terato- genic effect on the fetus, including increasing the incidence of neural tube defects. Choice D (Acetaminophen [Tylenol]) is incorrect. Acetaminophen (brand name Tylenol) is an analgesic (or pain reliever) and antipyretic (or fever reducer). Exposure to Acetaminophen, during the first month of pregnancy, has been shown to have a teratogenic effect on the fetus, includ- ing increasing the incidence of neural tube defects. Choice E (Famotidine [Pepcid]) is incorrect. Famotidine (brand name Pepcid) is a histamine H2-receptor antagonist that inhibits stomach acid production, and it is often used to treat gastroe- sophageal reflux disease (GERD). Exposure to Famotidine, during the first month of pregnancy, has been shown to have a teratogenic effect on the fetus, including increasing the inci- dence of neural tube defects.

A 19-year-old girl gives birth to a baby girl diagnosed with a meningomyelocele, as seen in the given midsagittal MRI. The mother lived in a rural area and did not have prenatal visits with her obstetrician. Which of the following drugs, if taken in the proper doses during the periconceptional period, would have greatly reduced the chances of her fetus having a neural tube defect (NTD)? (A) Valproic acid (Depakote) (B) Folic acid (Folacin) (C) Cephalexin(Keflex) (D) Acetaminophen(Tylenol) (E) Famotidine(Pepcid)

B: Burst fracture of T12 vertebral body. The sagittal CT clearly shows a burst (crush or compression) fracture of the 12th thoracic vertebral body (T12). This T12 burst fracture also presents with a fracture fragment that is displaced posterior into the vertebral canal where it would compress the terminal end of the spinal cord, the medul- lary cone (conus medullaris), which typically lies within the T12-L3 vertebral levels. Compression of the spinal cord would lead to pain within the lower back that would likely extend into the lower limbs. This injury might also result in incon- tinence, loss of sensation, and paraplegia or paraparesis (loss of motor function). Remember that a CT scan is a valuable means of viewing "blood and bone," and this type of radiologi- cal imaging is the standard way to view injuries resulting from trauma, as seen in this patient. With this patient falling off the ladder and landing on his feet, the downward force from the high fall and the upward force from the impact on the ground have crushed the body of the T12 vertebra, resulting in the burst fracture of the T12 vertebral body seen on this sagittal CT. Choice A (Herniated intervertebral disc) is incorrect. The intervertebral discs in a young person are usually so strong that the vertebrae often fracture before the discs rupture, and in this 22-year-old man, this scenario is confirmed by the sag- ittal CT scan. A herniated nucleus pulposus of the interverte- bral disc can cause lower back pain that extends into the lower limbs, as seen in this patient. However, in this CT scan, the spaces occupied by the discs, located between the vertebral bodies, are relatively uniform in both the lumbar and lower thoracic regions, which would also rule out a herniated inter- vertebral disc. A CT scan is valuable in assessing bone injuries while a MRI would be the best means to view damage to the intervertebral discs, the spinal cord and its nerve roots, and the ligaments of the spine. Choice C (Lumbar spinal stenosis) is incorrect. Lumbar spinal stenosis describes a narrowing of the vertebral foramen or canal that is normally seen in older individuals with a genetic disposition toward this condition. In the given CT, the vertebral canal, which houses the spinal cord, is relatively uniform throughout the lumbar region. The fracture fragment from the T12 vertebral body does enter the spinal canal and impinges the spinal cord and its nerve roots. However, lumbar spinal stenosis is usually due to degenera- tion of the intervertebral discs, which narrows the vertebral canal and potentially impinges the spinal cord and its nerve roots. Due to the age of the patient and the traumatic nature of his injury, lumbar spinal stenosis is not likely, especially with the visual evidence of a wide vertebral canal on the given CT scan. Choice D (Spondylolisthesis of L5-S1 articulation) is incorrect. Spondylolisthesis of the L5-S1 articulation is defined by the forward displacement of the vertebral body of L5 rela- tive to the upper part of the sacrum (S1). The given CT does not show spondylolisthesis of the L5-S1 articulation, so this choice can be eliminated. Choice E (Excessive lumbar lordo- sis) is incorrect. The natural curvature of the lumbar portion of the vertebral column is concave posterior, or lordosis. The secondary curvature of the lumbar spine to lordosis occurs when an infant begins to assume the upright position during standing or walking. The given CT does not show excessive lumbar lordosis (sway back or hollow back), which is often associated with weakened anterolateral abdominal muscula- ture, which is unlikely in this 22-year-old man.

A 22-year-old man is brought to the ER with severe back pain and lower limb weakness after falling off the top of a 16-ft ladder and landing on his feet. The ER physician orders a CT scan, and the given midsagittal CT directs the physician toward making which of the following diagnoses? (A) Herniated intervertebral disc (B) Burst fracture of T12 vertebral body (C) Lumbar spinal stenosis (D) Spondylolisthesis of L5-S1 articulation (E) Excessive lumbar lordosis

The answer is E: Pericardium. Due to the location of the stab wound, the pericardial sac and the right ventricle of the heart (the most anterior chamber of the heart) are most likely dam- aged. The pericardium is a fibrous, unyielding sac that sur- rounds the heart. Lesion of the right ventricle would cause blood to spill out of the heart into the pericardial space, caus- ing a pericardial effusion (blood accumulation within the peri- cardial sac). Compression of the heart (cardiac tamponade) would result. These events make it impossible for the heart to fill completely, which limits the blood it can receive. Patients with cardiac tamponade present with engorged veins of the face and neck due the backup of blood. A pericardial effusion can be drained via a procedure known as pericardiocentesis, which temporarily relieves the patient's problem until the heart wall is repaired. However, time is critical because cardiac tamponade compresses the heart and may result in a quick death. Choice A (Left lung) is incorrect. The left lung, and the pleural sac in which it resides, are pushed laterally away from the sternum due to the position of the heart, which resides primarily on the left side of the body. Due to the location of the knife wound in this patient, the pericardium and right ven- tricle would be damaged, not the left lung. Choice B (Pulmo- nary trunk) is incorrect. The pulmonary trunk is the outflow tract of the right ventricle. The pulmonary trunk resides left of the sternum at approximately the 2nd intercostal space, which is located above the entry point of the knife in this patient. Choice C (Left bronchus) is incorrect. The bifurcation of the trachea occurs at the 4th thoracic vertebral level. The wound location is at the 7th or 8th thoracic vertebral level, or the 5th intercostal space of the anterior thoracic wall. Therefore, the left main stem bronchus is located above the damaged area. Furthermore, the left bronchus resides deep to the heart, and the depth of the stab wound (~4 cm) would not reach the left bronchus. Choice D (Stomach) is incorrect. The stomach resides inferior to the location of the knife wound. Also, due to its posterior positioning and the depth of the wound, the stomach would not be damaged in this patient.

A 23-year-old man is stabbed in a bar fight. The blade of the knife enters his chest in the left 5th intercostal space, just lat- eral to the sternum, and pierces to a depth of approximately 4 cm. What structure is most likely damaged? (A) Leftlung (B) Pulmonarytrunk (C) Leftbronchus (D) Stomach (E) Pericardium

B: The pedicles of L3 and L4 are compressed together. Spinal nerves exit the vertebral canal through the intervertebral foramina on the lateral sides of the vertebral column. Each intervertebral foramen is formed by the juxtaposing of the pedicles of successive articulated vertebrae. Thus, if the pedicles are compressed together, the interver- tebral foramen is narrowed and a spinal nerve may be com- pressed ("pinched") as it traverses the intervertebral foramen. In the thoracic, lumbar, and sacral regions, each numbered segmental nerve passes through the intervertebral foramen below the matched numbered vertebra. Thus, the L3 spinal nerve traverses the intervertebral foramen below the L3 ver- tebra, between the L3 and L4 vertebrae. Because there is one more cervical spinal nerve (N = 8) than cervical vertebrae (N = 7), the cervical nerves exit the intervertebral foramina above the matching numbered vertebrae. Thus, the C3 spinal nerve passes through the opening between vertebrae C2 and C3. In this patient, the damage to the L3 spinal nerve on the left side is due to narrowing the intervertebral foramen caused by the compression of the pedicles of L2 and L3. Choice A (The pedicles of L2 and L3 are compressed together) is incor- rect. When the pedicles of L2 and L3 are compressed together, the result is stenosis of the L2 intervertebral foramen. This nar- rowing would damage the ipsilateral L2 spinal nerve, result- ing in lower back pain and potentially lower limb deficits. In this patient, damage to the L3 spinal nerve on the left side was reported, so compression of the L2 and L3 pedicles is not likely. Choice C (The laminae of L2 and L3 are compressed together) is incorrect. Compression of the interlaminar spaces would not affect the spinal nerves, as they do not pass through these openings. In fact, such compression would cause a lever arm reaction that would expand the intervertebral foram- ina. Choice D (The laminae of L3 and L4 are compressed together) is incorrect. Compression of the interlaminar spaces would not affect the spinal nerves, as they do not pass through these openings. In fact, such compression would cause a lever arm reaction that would expand the intervertebral foramina. Choice E (The spinous processes of L3 and L4 are compressed together) is incorrect. Compression of the spinous processes does not affect the spinal nerves because they do not pass through these gaps. Again, a compression of the L3 and L4 spinous processes would expand the intervertebral foramina because of a lever arm reaction.

A 25-year-old medical student feels a sharp pain in his lower back while helping his landlady move a piano. After 3 days, the pain does not subside, so he receives a comprehensive examination from his physician that reveals a compression of the left side L3 spinal nerve. Which of the following is the most likely cause of this condition? (A) The pedicles of L2 and L3 are compressed together (B) The pedicles of L3 and L4 are compressed together (C) The laminae of L2 and L3 are compressed together (D) The laminae of L3 and L4 are compressed together (E) The spinous processes of L3 and L4 are compressed together

B: Paraplegia and incontinence. Transection of the spinal cord results in loss of all motor and sensory functions movements that emerge from the cord inferior to the site of the lesion. General sensory, somatic motor, and visceral motor fibers for the pelvis and lower limbs would be lost inferior to the level of T9 due to the transection of the spinal cord. Also, parasympathetic outflow to the pelvis and perineum would be eliminated due to the origin of these nerve fibers from S2-4. The CT image reveals a T9 spinal fracture dislocation, and the clinical findings indicate a spinal cord transection. Therefore, this patient would present with paraplegia (or paralysis of both lower limbs), fecal and urinary incontinence, impotence, and loss of motor functions and sensation below the level of the T9 lesion, which is right above the level of the umbili- cus (supplied by T10). Choice A (Quadriplegia and inconti- nence) is incorrect. A transection of the spinal cord at the level of T9 would spare the upper limbs from sensory and motor deficits because the upper limbs are supplied by the anterior rami of C5-T1. This patient would present with paraplegia (or paralysis of both lower limbs), fecal and urinary inconti- nence, impotence, and loss of motor functions and sensory input below the level of the T9 lesion. Choice C (Incontinence only) is incorrect. A transection of the spinal cord at the level of T9 would result in paraplegia (or paralysis of both lower limbs), fecal and urinary incontinence, impotence, and loss 11 of motor functions and sensory input below the level of the T9 lesion. Though incontinence would result from this spinal cord transection at T9, this deficit would be accompanied by additional signs and symptoms due to the severity and loca- tion of this spinal cord injury. Choice D (Loss of only sensory information below the lesion) is incorrect. Transection of the spinal cord at T9 would result in the loss of all sensation and voluntary movements inferior to T9. Therefore, loss of only sensory information is not a viable option. Choice E (Loss of only motor function below the lesion) is incorrect. Transection of the spinal cord at T9 would result in the loss of all sensation and voluntary movements inferior to T9. Therefore, loss of only motor function is not a viable option in this patient.

A 34-year-old man fell 25 ft out of a barn loft, landing on his back. He was found unconscious and taken to the ER. The given sagittal CT reformat image reveals a T9 spi- nal fracture dislocation, as noted by the black arrow. The patient had unequivocal clinical findings indicating spi- nal cord transection. Given the results of this CT image and the clinical information, what deficits will the patient incur? (A) Quadriplegia and incontinence (B) Paraplegia and incontinence (C) Incontinenceonly (D) Loss of only sensory information below the lesion (E) Loss of only motor function below the lesion

s D: Flexing the hip by raising the straightened left leg with the patient lying in the supine position increases pain in the back with radiation down the left leg. The straight-leg raising test is designed to stretch the lower back and apply tension to the lumbar spinal nerves and roots. Thus, a posi- tive test will stress compromised nerves and elicit pain and/ or spasticity in the areas supplied by the affected nerves. If the diagnosis of left side lumbar nerve entrapment second- ary to a herniated lumbar disc is correct, raising the left leg (i.e., flexing the hip) with the knee straight (extended) from the supine position (lying on the back) will apply traction to the entrapped nerve roots and elicit pain in the affected areas. Conducting the same maneuver on the right side will not elicit a pain reaction. Lasèque sign is a medical maneuver that involves raising a straight leg to exacerbate pain and is useful in diagnosing lumbar disc disorders and tension of the sciatic nerve. Choice A (Walking while keeping the left leg in a straight position [i.e., the knee is kept straight] relieves the pain in the back and limb) is incorrect. The forward swing of the straight left leg during walking is essentially the same maneuver as raising the straight leg from the supine position. Traction is applied to the entrapped nerve roots during flexion of the hip, with pain stimulated rather than relieved. Choice B (Flexing the hip by raising the straightened left leg [i.e., the knee is kept straight] with the patient lying in the supine position relieves the pain in the back and leg) is incorrect. This maneuver is basically the same as the one explained in Choice A. Again, pain should be elicited rather than relieved in a positive test. Choice C (Abducting the hip by raising the straightened left leg with the patient lying on his right side relieves the pain in the back and leg) is incorrect. Raising (abducting) the limb toward the affected side will again apply traction to the affected nerve roots, resulting in stimulating pain rather than relieving pain. Choice E (Extending the hip by raising the straightened left leg with the patient lying in the prone position increases pain in the back with radiation down the left leg) is incorrect. From the prone position, rais- ing the left limb means extending the limb. This maneuver will not produce as great an excursion of the limb as flex- ing the straightened limb at the hip from the supine position. Therefore, this maneuver will reduce traction on the affected nerve roots and relieve pain rather than increase pain.

A 35-year-old man presents with pain radiating from his lower back into his left lower limb, periodic sensory loss and par- esthesia in that limb, and associated motor weakness. His physician suspects unilateral entrapment of lumbar nerve roots secondary to a herniated lumbar intervertebral disc. In examining the patient, the physician conducts a straight-leg raising test. Which of the following best describes a positive straight-leg raising sign (Lasèque's sign) for this condition in this patient? (A) Walking while keeping the left leg in a straight position (i.e., the knee is kept straight) relieves the pain in the back and limb (B) Flexing the hip by raising the straightened left leg (i.e., the knee is kept straight) with the patient lying in the supine position relieves the pain in the back and leg (C) Abducting the hip by raising the straightened left leg with the patient lying on his right side relieves the pain in the back and leg (D) Flexing the hip by raising the straightened left leg with the patient lying in the supine position increases pain in the back with radiation down the left leg (E) Extending the hip by raising the straightened left leg with the patient lying in the prone position increases pain in the back with radiation down the left leg

A: Posterior (dorsal) roots. A rhizotomy is a neurosurgical procedure that selectively severs problematic spinal nerve roots to relieve pain or spastic paralysis (e.g., as often seen in cerebral palsy patients). The posterior roots are the only site where afferent (sensory) fibers are segregated from efferent (motor) fibers. In selective dorsal rhizotomy (SDR), severing the left C6-8 posterior roots could relieve the pain symptoms in the left forearm of this patient because they carry only sensory information. Choice B (Posterior [dorsal] primary rami) is incorrect. Cutting the posterior primary rami of spinal nerves would cause degeneration of the distal axonal processes of the general sensory (afferent) fibers as well as fibers to the intrinsic back muscles. However, the general sen- sory fibers contained within the posterior primary rami only supply innervation to the skin over the back, so the patient's pain within the left forearm would remain. Because they do not convey pain fibers from the left forearm, selective rhizotomy of the posterior primary rami would not relieve this patient's pain. Choice C (Spinal nerves) is incorrect. Cutting the spi- nal nerves would cause loss of sensory, motor, and (depend- ing on the vertebral level) autonomic deficits. Because spinal nerves convey both sensory and motor fibers, this location is inappropriate for selective rhizotomy to relieve pain in the left forearm. Choice D (Anterior [ventral] roots) is incorrect. The anterior roots of the spinal nerve are the only site where motor fibers are segregated from sensory fibers. So, selective rhizotomy at the anterior roots would cause motor deficits but would not relieve the patient's pain. Choice E (Anterior [ventral] primary rami) is incorrect. Cutting the appropriate anterior primary rami of spinal nerves, which supply the left forearm, would cause degeneration of the distal axonal pro- cesses of the general sensory (afferent) fibers and relieve pain symptoms in this patient. However, the anterior primary rami also contain somatic motor (general somatic efferent or GSE) fibers, so a selective rhizotomy at this location would also cause motor deficits. Because they convey both sensory and motor fibers, the anterior primary rami are inappropriate for selective rhizotomy to afford relief from pain.

A 35-year-old woman suffers intractable pain in her left fore- arm. Neurosurgical consultation leads to a decision to conduct a rhizotomy to relieve the condition. At which of the following locations is the rhizotomy best performed to relieve the patient's pain? (A) Posterior (dorsal) roots (B) Posterior (dorsal) primary rami (C) Spinalnerves (D) Anterior (ventral) roots (E) Anterior (ventral) primary rami

C: Spina bifida occulta. Spina bifida occulta is the mildest form of spina bifida (L: split spine). In this devel- opmental disorder, the left and right neural arch elements fail to fuse completely in the dorsal midline. However, the split in the vertebra is so small that the meninges and elements of the spinal cord do not protrude through the defect. In the given plain film, the black arrow indicates a failure of fusion of the laminae of L5, producing a cleft. Individuals with spina bifida occulta may possess a tuft of hair and/or dimple in the skin overlying the affected vertebral levels, as noted in this patient. Other individuals with this condition may have no visible evidence, a lipoma, or even a birthmark in the overly- ing skin of the affected region. Some studies suggest approxi- mately 10% of the general population have this mildest form of spina bifida. In this case, the X-ray verifies the diagnosis of spina bifida occulta. Because this condition is normally clinically asymptomatic and unnoticed, it is seemingly "hid- den" (occult). In fact, most research suggests no relationship between spina bifida occulta and back pain. Choice A (Menin- gocele) is incorrect. The least common form of spina bifida is a posterior meningocele (or meningeal cyst). In this develop- mental disorder, the bilateral neural arch elements fail to fuse completely in the dorsal midline but the meninges protrude through the defect into a sac or cyst. Therefore, multiple ver- tebral defects are present, accompanied with the presence of a cyst, which contains cerebrospinal fluid (CSF). In a menin- gocele, the spinal cord and nerve roots are typically in normal position, not protruding into the cyst, and there are usually no long-term effects on the individuals. Without the presence of a meningeal cyst in the lower back of this man, a menin- gocele can be easily eliminated. Choice B (Anencephaly) is incorrect. Anencephaly is a severe neural tube defect in which the cephalic (head) end of the neural tube fails to close in the embryo. This birth defect is lethal with the baby being born without a forebrain, skullcap (calvaria), and scalp, leaving the remaining portions of the brain exposed. If proper prenatal care is available, most cases of anencephaly are detected by elevated maternal serum alpha-fetoprotein (AFP) levels and ultrasound examinations during prenatal examinations. Ele- vated AFP levels are often correlated with neural tube defects. The addition of folic acid to the diet of women in their child- bearing years has been shown to reduce the incidence of neural tube anomalies. A patient with anencephaly would not reach 45 years of age. Choice D (Spina bifida cystica) is incorrect. Spina bifida cystica is a severe form of spina bifida and receives its name because of the characteristic presence of a cyst-like sac protruding from the defective area. The membranous walls of the sac are composed of very thin skin plus dura and arach- noid components. In these neural tube defects, the unfused neural arches of multiple vertebrae allow the meninges (and potentially the spinal cord) to protrude through the structural defect. The inclusion of a displaced portion of the spinal cord and nerve roots in the malformation designate this neural tube defect as a meningomyelocele, which is the most common form of spina bifida cystica. That is, the defect involves the meninges (meningo-), the spinal cord (myelo-) and the mem- branous sac (-cele). Because of the severity of this type of spina bifida, this 45-year-old man would not have this condition. Choice E (Spina bifida with myeloschisis) is incorrect. Spina bifida with myeloschisis, or rachischisis, involves the same signs and symptoms as the most common form of spina bifida cystica, a meningomyelocele; however, the protruded portions of the spinal cord are not afforded the enveloping protection of the meninges. Physical examination would reveal a flattened mass of nervous tissue with no associated membranes, which makes the patient more prone to life-threatening infections. Because of the severity of spina bifida with myeloschisis, this 45-year-old man would not have this condition.

A 45-year-old man goes to his family physician complaining of lower back pain after spending a weekend clearing trees off his property. During examination, the doctor notes a tuft of hair and a dimple on the skin of the patient's lower back. A plain film reveals a congenital defect in his L5 vertebra, indicated by the black arrow in the given X-ray. No other structural abnor- malities are noted. Based on these findings, what is the most likely diagnosis for this patient? (A) Meningocele (B) Anencephaly (C) Spinabifidaocculta (D) Spinabifidacystica (E) Spina bifida with myeloschisis

C: Jefferson (burst) fracture of C1. The C1 vertebra, or atlas, is normally a closed ring with no vertebral body. Excessive vertical, or downward, force on the top of the head can fracture the anterior and posterior arches of C1 in multiple places, leading to a Jefferson (burst) fracture of C1. Due to the vertical force of the concrete mix striking the top of the man's head, the lateral masses of C1 are driven laterally due to the oblique articulation between the occipital condyles and the superior articular processes of the lateral masses of C1. This vertical compression force fractures the anterior and posterior arches of C1 bilaterally, as confirmed by the axial CT scan. Jefferson fractures of C1 often occur with axial loading force when the top of the head is impacted by a hard or heavy object. Because the fractures within the bony ring of the atlas actually increase its dimension, this type of fracture does not usually result in spinal cord injury; however, upper neck pain would be present. The axial CT scan confirms the diagnosis of a Jefferson (burst) fracture of C1. Choice A (Damage to the cervical spinal cord) is incorrect. Damage of the cervical spinal cord is highly unlikely due to the lack of neurological signs reported in this patient. The axial CT scan depicts bilateral fractures within the anterior and posterior arches of C1, and none of these fracture fragments are in close approximation to the spinal cord. Choice B (No pathology is apparent on the CT scan) is incorrect. The axial CT scan depicts bilateral fractures within the anterior and posterior arches of C1. Knowledge of the anatomy of the atlas (C1) would eliminate this diagnosis as a likely option. Choice D (Fracture of the dens axis [odon- toid process]) is incorrect. In the given axial CT scan of the C1 arch, it would be hard to detect a fracture of the dens axis (odontoid process). Moreover, odontoid fractures have been reported to occur following a combination of excessive flex- ion, extension, and some rotation within the cervical verte- brae, so the nature of this traumatic injury would not sug- gest an odontoid fracture. A sagittal CT scan would give more information concerning the integrity of the odontoid process. Choice E (Atlanto-axial subluxation) is incorrect. Atlanto- axial subluxation, or the incomplete dislocation of the median atlanto-axial joint, occurs following the rupture of the trans- verse ligament of the atlas, which holds the dens axis (odon- toid process) in place. Losing the integrity of the transverse ligament of the atlas can result in compression of the upper cervical spinal cord, leading to quadriplegia (paralysis of all four limbs) and even death (if the medulla of the brainstem is compressed). In this axial CT scan, the odontoid process (dens axis) of C2 is in its normal position, which implies an intact transverse ligament of the atlas. Therefore, atlanto-axial subluxation did not occur in this patient.

A 46-year-old supervisor was reading a work order on a con- struction site when a 60 lb bag of concrete mix was acciden- tally dropped on the apex of his head. He was immobilized and brought to the ER where he presented with upper neck pain but no neurological signs. Based upon the given axial CT scan and the patient's presentation, which of the following diagnoses is most likely? (A) Damage to the cervical spinal cord (B) No pathology is apparent on the CT scan (C) Jefferson (burst) fracture of C1 (D) Fracture of the dens axis (odontoid process) (E) Atlanto-axialsubluxation

C: Fibrous. Fibrous cartilage, or fibrocar- tilage, is one of the three types of cartilage composed of chondrocytes that secrete, and are surrounded by, an exten- sive extracellular matrix composed of collagen fibers, elastin fibers, and a plethora of ground substance rich in proteo- glycan. Fibrocartilage is a very tough, white material that provides high tensile strength to the anulus fibrosus of the IV discs, which would have ruptured in this patient. The fibrocartilage of the anulus fibrosus is composed of a dense network of Type I collagen, but it can tear due to sudden hyperflexion in older individuals (such as when lowering heavy objects), which pushes the gelatinous nucleus pul- posus of the IV disc posterior toward the thinnest part of the anulus fibrosus. When the fibrocartilage of the anulus fibrosus is compromised, the nucleus pulposus may her- niate into the vertebral canal and compress spinal nerve roots or the spinal cord, leading to back pain. Fibrocar- tilage, or fibrous cartilage, provides the tensile strength of the IV discs. Choice A (Hyaline) is incorrect. Hyaline cartilage is one of the three types of cartilage composed of chondrocytes that secrete, and are surrounded by, an extensive extracellular matrix composed of collagen fibers, elastin fibers, and a plethora of ground substance rich in proteoglycan. Hyaline cartilage is a translucent, hard mate- rial rich in Type II collagen fibers and proteoglycan. Cov- ering the articular surfaces of bones, it is found in many joints. Hyaline cartilage is also found in the tracheal rings, larynx, and costal cartilages. In endochondral ossification, bone is formed from a hyaline cartilage intermediate. It is also found in maturing long bones within the epiphysial (or growth) plates. Hyaline cartilage does not provide the tensile strength of the IV discs. Choice B (Elastic) is incor- rect. Elastic cartilage is one of the three types of cartilage composed of chondrocytes that secrete, and are surrounded by, an extensive extracellular matrix composed of collagen fibers, elastin fibers, and a plethora of ground substance rich in proteoglycan. It is composed of large amounts of elas- tic fibers and Type II collagen, and it is found in the exter- nal ear (auricle), pharyngotympanic (auditory) tube, and epiglottis, where it provides a rigid but elastic framework. Elastic cartilage does not provide the tensile strength of the IV discs. Choice D (Epiphysial) is incorrect. In endochon- dral ossification, bone is formed from a hyaline cartilage intermediate. It is also found in maturing long bones and is called epiphysial cartilage, located within the epiphysial (growth) plates of bone. This new type of cartilage forms the resting zone of chondrocytes located on the epiphysial (distal) side of the zone of growing cartilage, which unites the epiphysis with the shaft of the long bone. Because the patient is 55 years old, the presence of epiphysial cartilage is unlikely. Moreover, fibrocartilage provides the tensile strength of the IV discs. Choice E (Anular) is incorrect. Anular cartilage is a synonym for the cricoid cartilage, the lowest portion of the laryngeal skeleton. Because the cricoid cartilage is shaped like a signet ring, with its arch located anterior and its widened lamina posterior, it is sometimes referred to as the anular cartilage. Because the anular carti- lage is not a type of cartilage, it is unable to provide tensile strength to the IV disc and can easily be eliminated as a possible answer. Fibrocartilage provides the tensile strength of the IV discs.

A 55-year-old man experiences severe lower back pain for 4 days after moving heavy furniture. After a thorough back evaluation and review of associated radiological imaging, a surgeon removes the herniated portion of the L3-4 interver- tebral (IV) disc. What type of cartilage most likely gives the tensile strength of the disc extracted from this patient? (A) Hyaline (B) Elastic (C) Fibrous (D) Epiphysial (E) Anular

D: Cervical Spinal Stenosis. Cervical spinal stenosis describes a narrowing of the vertebral canal that is typically seen in older individuals with degenerative changes 21 in the cervical spine. In the given MRI, the vertebral canal, which houses the spinal cord, is impinged by intervertebral disc degeneration in the midcervical region, which results in no cerebral spinal fluid (CSF), which appears as a hyperin- tense (white) signal in this MRI, being visible anterior or pos- terior to the spinal cord. CSF can be seen surrounding the spinal cord above and below the midcervical region. In this patient, the tingling numbness of the fourth and fifth digits within his right hand is exacerbated by hyperextension of the cervical vertebrae (or looking up), which implies the spinal cord is further compressed by hyperextension of the neck leading to myelopathy (damage to the spinal cord itself). Due to the age of the patient, the degenerative nature of the cervi- cal vertebrae, and the absence of trauma, spinal cord stenosis is the most likely diagnosis, and this condition was confirmed with the given T2-weighted MRI. Choice A (Burst fracture of C3 vertebral body) is incorrect. A burst (crush or compres- sion) fracture of the vertebral body of the 3rd cervical vertebra (C3) would likely result from trauma, and no trauma has been reported in this patient. Moreover, the given MRI does not confirm a burst fracture of C3, though degenerative changes can be seen within this vertebra. A burst fracture of C3 could lead to compression of the spinal cord due to a displaced posterior fracture fragment; however, the given MRI does not confirm this type of injury. Choice B (Ruptured anterior longitudinal ligament) is incorrect. The anterior longitudinal ligament is a vertically running band that attaches along the anterior sides of the vertebral bodies, which resists hyper- extension of the vertebral column. Damage to the anterior longitudinal ligament can occur following traumatic hyper- extension of the head on the neck, which was not reported in this patient. Moreover, damage to (or interruption of) the anterior longitudinal ligament would appear as a hyperintense (white) signal located anterior to the vertebral column at the location of the damage to this ligament, and this evidence is not apparent on this T2-weighted MRI. Choice C (Transec- tion of the spinal cord) is incorrect. Transection of the spinal cord would result in the loss of all sensation and voluntary movements inferior to the site of the lesion. Visual evidence of a transection of the cervical spinal cord is not apparent on this MRI, and because the patient only reported a tingling numb- ness of the fourth and fifth digits within his right hand, this diagnosis is not possible. Choice E (Traumatic spondylolis- thesis of C5) is incorrect. Traumatic spondylolisthesis of C5 describes the anterior displacement of the C5 vertebral body in relation to the C6 vertebra located below it. In this patient, spondylolisthesis of C5 is apparent on the sagittal MRI. How- ever, due to the absence of trauma, the anterior displacement of C5 is probably due to degenerative changes (such as facet arthritis, disc degeneration, and the presence of osteophytes, or bone spurs) within the cervical vertebrae. This radiologic evidence would imply degenerative spondylolisthesis, not traumatic spondylolisthesis, of C5 as the reason for the ante- rior displacement of C5.

A 73-year-old man comes to his physician complaining of a tingling numbness within the fourth and fifth digits of his right hand. These symptoms are exacerbated when he looks up at the ceiling. The given sagittal T2-weighted MRI reveals degenerative changes in his cervical vertebrae. Which of the following diagnoses is confirmed by the MRI and would result in the symptoms of this patient? (A) Burst fracture of C3 vertebral body (B) Ruptured anterior longitudinal ligament (C) Transection of the cervical spinal cord (D) Cervical spinal stenosis (E) Traumatic spondylolisthesis of C5

The answer is D: Pulmonary infundibular stenosis. An abnor- mally narrowed right ventricular outflow tract (pulmonary infundibular stenosis) is one of the four components of TOF. This syndrome is the most common malformation complex resulting from unequal division of the conus cordis and truncus arteriosus by the spiraling conotruncal septum. This septum contributes significantly to the formation of the ven- tricular outflow tracts and the proximal parts of the aorta and pulmonary trunk. Anterior displacement of the developing conotruncal septum results in a narrow right ventricular out- flow (pulmonary infundibular stenosis), plus an overriding aorta, ventricular septal defect (VSD), and hypertrophied right ventricle. The four classic characteristics of TOF are pictured in the given figure. Children with TOF are typically cyanotic due to the mixing of right and left side blood through the VSD and the overriding aorta. Also, the systolic heart mur- mur is typical due to the VSD. Choice A (Transposition of the great vessels) is incorrect. Failure of the conotruncal septum to follow its normal spiral course results in transposition of the great vessels. When the septum runs directly downward through the conotruncal region instead of spiraling, the aorta originates from the right ventricle and the pulmonary trunk arises from the left ventricle. Choice B (Hypertrophy of the left ventricle) is incorrect. In TOF, the combination of back- pressure from the infundibular stenosis and systemic pressure needs from the overriding aorta and VSD results in hyper- trophy of the right ventricle. Choice C (Interatrial septal defect) is incorrect. The conotruncal septum contributes to the formation of the upper (membranous) part of the inter- ventricular septum. Thus, malformation of the conus septum commonly results in a membranous VSD, not an atrial septal defect. Choice E (Aortic valvular atresia) is incorrect. In this condition, the valvular orifice into the aorta is absent, and the aorta and left side chambers are underdeveloped. In TOF, the conotruncal septum is displaced, resulting in an open, overriding aorta.

A child is born at home without difficulty. Two weeks later, the mother takes the infant to her doctor, reporting that he "turns blue" when he cries. Physical examination reveals the infant is cyanotic and has a distinct systolic heart murmur. The physician suspects the baby has a tetralogy of Fallot (TOF). Which of the following conditions is a component of this syndrome? (A) Transposition of the great vessels (B) Hypertrophy of the left ventricle (C) Interatrial septal defect (D) Pulmonary infundibular stenosis (E) Aortic valvular atresia

D: Longissimus. Destruction of the embryonic epimeres will result in loss of the deep group of back muscles, including the longissimus muscle, one of three erector spinae muscles (along with the iliocostalis and spinalis muscles). Muscles derived from the epimeres are the proper muscles of the back, which are innervated by the posterior (dorsal) pri- mary rami of spinal nerves. These deep muscles of the back include the previously mentioned erector spinae group, sple- nius capitis and cervicis muscles, suboccipital muscles, trans- versospinales muscles (including the semispinalis, multifidus, and rotatores muscles), and other small muscles. Remember, each embryonic somite differentiates into three components: myotome, dermatome, and sclerotome. Each myotome splits into two parts: epimere and hypomere. The epimere devel- ops dorsal to the incipient vertebral column, and the long- issimus muscle is the only listed muscle which is derived from the embryonic epimere. The muscles derived from the epimeres form the deep group of muscles in the back and are often termed the intrinsic back muscles or epaxial muscles. Choice A (Trapezius) is incorrect. The trapezius is a member of the superficial extrinsic layer of back muscles, which connect the upper limbs to the trunk. When these muscles contract (or shorten), they produce movements of the upper limb. How- ever, the trapezius muscle does not originate from either the epimere or hypomere, as evidenced by its innervation via the spinal accessory nerve (CN XI). Instead, it is likely related to postbranchial origins, along with its mate, sternocleidomas- toid. Thus, destruction of epimeres will not directly affect the formation of the trapezius. Choice B (Latissimus dorsi) is incorrect. The latissimus dorsi (L: widest muscle of back) is a member of the superficial extrinsic layer of back muscles, which connect the upper limbs to the trunk. When these muscles contract, they produce movements of the upper limb. The latissimus dorsi muscle is derived from hypom- eres, which develop lateral and anterior to the vertebral axis, and it migrates secondarily into the back. All of the muscles derived from hypomeres are innervated by the anterior (ven- tral) primary rami of spinal nerves. The latissimus dorsi muscle is innervated by the thoracodorsal (middle subscapu- lar) nerve and acts on the upper limb. Due to being derived from the embryonic hypomeres, the latissimus dorsi muscle would not be affected by the toxin injected into the epim- eres within this mouse embryo. Choice C (Rhomboid major) is incorrect. The rhomboid major muscle is a member of the superficial extrinsic layer of back muscles, which connect the upper limbs to the trunk. When these muscles contract, they produce movements of the upper limb. The rhomboid major muscle is derived from hypomeres, which develop lateral and anterior to the vertebral axis. All of the muscles derived from hypomeres are innervated by the anterior (ventral) primary rami of spinal nerves. Due to being derived from the embry- onic hypomeres, the rhomboid major muscle would not be affected by the toxin injected into the epimeres within this mouse embryo. Choice E (Levator scapulae) is incorrect. The levator scapulae muscle is a member of the superficial extrinsic layer of back muscles, which connect the upper limbs to the trunk. When these muscles contract, they pro- duce movements of the upper limb. The levator scapulae muscle is derived from hypomeres, which develop lateral and anterior to the vertebral axis. All of the muscles derived from hypomeres are innervated by the anterior (ventral) primary rami of spinal nerves. Due to being derived from the embry- onic hypomeres, the levator scapulae muscle would not be affected by the toxin injected into the epimeres within this mouse embryo.

A developmental biologist studying the derivatives of the somites and somitomeres treats the epimeres in a mouse embryo with a toxin that kills all of their cells. If the mouse develops to full term, which of the following muscles is most likely to be absent? (A) Trapezius (B) Latissimusdorsi (C) Rhomboidmajor (D) Longissimus (E) Levatorscapulae

E: Posterior (dorsal) primary ramus. Sensory receptors in the skin overlying the trapezius muscle project through general sensory (general somatic afferent) neurons to the spinal cord via the posterior (dorsal) primary rami of spi- nal nerves (marked "E" in this diagram), traverse the mixed spinal nerves, travel within posterior (dorsal) roots of spinal nerves, and reach the posterior (dorsal) gray horn of the spi- nal cord. Cutting the posterior rami of spinal nerves would cause degeneration of the distal axonal processes of the gen- eral sensory fibers and lead to loss of sensation in the skin of the back. Additionally, the distal axonal processes of somatic motor (general somatic efferent or GSE) neurons and visceral motor (general visceral efferent or GVE) neurons contained within the posterior primary rami would be damaged as well, causing motor and autonomic deficits in the back, respectively. Remember that the trapezius muscle is a component of the superficial extrinsic layer of back muscles, which connect the upper limbs to the trunk. These muscles are innervated by anterior primary rami of spinal nerves, except for the trape- zius, which is supplied by the accessory nerve (CN XI). How- ever, the skin overlying the trapezius muscle is innervated by the posterior (dorsal) primary rami of spinal nerves. Choice A (Lateral gray horn of spinal cord) is incorrect. The lateral gray horn of the spinal cord is the location of the intermediolateral cell column (IML), which contains the cell bodies of presynap- tic (preganglionic) sympathetic neurons of the autonomic ner- vous system (ANS). The IML only exists within the lateral gray horn between spinal segmental levels T1-L2 (or L3), which is the reason the sympathetic division of the ANS is also called the thoracolumbar division of the ANS. A lesion of the lateral gray horn of the spinal cord would cause autonomic deficits; however, it would not affect the general sensory fibers supply- ing cutaneous innervation to the skin overlying the trapezius muscle. Choice B (Anterior root of spinal nerve) is incorrect. The anterior (ventral) roots of spinal nerves convey the axonal processes of somatic and visceral motor neurons into the periphery. Damage to the anterior roots of spinal nerves would not affect the general sensory fibers supplying cutaneous inner- vation to the skin overlying the trapezius muscle. However, damage to the anterior roots of spinal nerves would affect mus- cles in the superficial layer of the back, except for the trapezius muscle, which is innervated by the accessory nerve (CN XI). Choice C (Sympathetic trunk) is incorrect. The sympathetic trunk carries both presynaptic and postsynaptic (postgangli- onic) sympathetic neurons as well as visceral sensory (general visceral afferent or GVA) neurons. The visceral sensory neu- rons within the sympathetic trunk convey pain from organs of the body cavities and are not involved with cutaneous inner- vation to the skin overlying the trapezius muscle. Choice D (Anterior primary ramus) is incorrect. The anterior (ventral) primary rami of spinal nerves contain general sensory, somatic 5 motor, and visceral motor fibers. However, the general sensory fibers in the anterior primary rami are bringing sensory infor- mation from the lateral and anterior parts of the body wall, including the limbs. Therefore, cutting the anterior primary rami would not affect innervation to the skin overlying the trapezius muscle. It would, however, affect the proprioceptive (general sensory) fibers leaving the trapezius muscle, which are carried by the anterior primary rami of C3 and C4.

A lesion of which of the indicated structures would cause loss of sensation in the skin overlying the trapezius muscle?

A: The medullary cone ends at or above the L3 level. The objective of a lumbar puncture is to enter (tap) the subarachnoid space and access the CSF. This procedure is performed using a long spinal needle. For several reasons, this procedure is best performed in the low lumbar region, between the spinous processes of the L3 and L4 (sometimes L4 and L5) vertebrae. The medullary cone (or conus medullaris) is the tapered terminal end of the spinal cord. In adults, the med- ullary cone is normally located within the T12-L3 vertebral levels. Thus, penetrating the vertebral canal and subarachnoid space below L3 is the "safe" place to go, in that the spinal needle should not penetrate the spinal cord. Choice B (The subarach- noid space ends at the L3 level) is incorrect. The subarachnoid space is the CSF-filled space between the arachnoid and pia mater layers of the meninges. The dura-arachnoid layers line the vertebral canal and end at the S2 level. The pia mater lines the exterior surface of the neural tube and continues distally to envelope the spinal nerve rootlets and roots. Because the spinal cord ends at approximately the L1 level, there is a large separation between the dura and arachnoid mater layers of the meninges and the pia mater below L1. Thus, the subarachnoid space is quite large below the medullary cone (conus medul- laris), providing a significant pool (the lumbar cistern) of CSF. Choice C (The intervertebral foramina at L3-4 are large and easy to penetrate) is incorrect. The intervertebral foramina in the lumbar region are large openings; however, the lumbar puncture does not occur at these locations. During a spinal tap, the spinal needle is inserted through an interlaminar space, on the posterior side of the vertebral column. These spaces are largest in the low lumbar spine, thus affording a relatively open path to the vertebral canal. Before a spinal tap is administered, the patient is often bent forward into the fetal position by flexing the spine, which expands the interlaminar spaces during the lumbar puncture. Choice D (No vertebral venous plexuses exist below the L3 level) is incorrect. The ver- tebral venous plexuses are an extensive network of valveless, interconnecting vessels running the entire length of the verte- bral column. The external vertebral venous plexus lies on the anterior aspect of the vertebral column and the posterior side of the vertebral arch. The internal vertebral venous plexus is a major constituent of the epidural space within the vertebral canal. A lumbar puncture must traverse both the external and internal plexuses to reach the subarachnoid space. Choice E (The ligamenta flava are absent below the L3 level) is incor- rect. The ligamenta flava are short ligaments that connect the anterior sides of adjacent vertebral laminae along the entire length of the vertebral column. The spinal needle must pierce a ligamentum flavum in order to enter the vertebral canal.

A physician orders a lumbar puncture (spinal tap) for his 43-year-old female patient in order to obtain a sample of cere- brospinal fluid (CSF). He explains to her that this procedure will be done in the lower back, between the spinous processes of the L3 and L4 vertebrae. What is the best reason for per- forming the lumbar puncture at this location? (A) The medullary cone ends at or above the L3 level (B) The subarachnoid space ends at the L3 level (C) The intervertebral foramina at L3-4 are large and easy to penetrate (D) No vertebral venous plexuses exist below the L3 level (E) The ligamenta flava are absent below the L3 level

A: Anterior gray horn of the spinal cord. Cell bodies of somatic motor neurons (a-motor neurons) innervat- ing skeletal muscle are located within the anterior (ventral) gray horn of the spinal cord, at all segmental levels through- out the entire length of the spinal cord. The innervation of the skeletal muscles affected by ALS is through somatic motor (general somatic efferent or GSE) neurons and bran- chial motor (special visceral efferent or SVE) neurons (neu- rons that supply the embryonic pharyngeal arches). In ALS patients, postmortem analysis of the anterior gray horn of the spinal cord would show significant degeneration. In the given diagram, the anterior gray horn of the spinal cord is labeled as "A." The locations for all five possible choices for this question are also indicated in this figure. Choice B (Lateral gray horn of the spinal cord) is incorrect. The lateral gray horn of the spinal cord is the location of the intermediolateral cell column (IML), which contains the cell bodies of presynaptic (preganglionic) sympathetic neurons of the autonomic nervous system (ANS). The IML only exists within the lateral gray horn between spinal segmental levels T1-L2 (or L3), which is the reason the sym- pathetic division of the ANS is also called the thoracolumbar division of the ANS. Degeneration in the lateral gray horn of the spinal cord would cause autonomic deficits but would not include the somatic motor neurons involved in ALS. Choice C (Posterior gray horn of the spinal cord) is incorrect. The pos- terior (dorsal) gray horn, located along the entire length of the spinal cord, contains cell bodies of spinal interneurons. The central process of each pseudounipolar sensory neuron conveys general sensory (afferent) fibers through the posterior (dorsal) nerve root and synapses within the posterior horn of the spinal cord. Degeneration within the posterior gray horn of the spinal cord would cause sensory deficits but would not include the somatic motor neurons involved in ALS. Choice D (Spinal ganglia) is incorrect. The spinal (dorsal root) ganglia, located at the distal ends of the posterior (dorsal) roots of spi- nal nerves, contain cell bodies of the general sensory (afferent) neurons. Degeneration within the spinal ganglia would result in sensory deficits but would not include the somatic motor neurons involved in ALS. Choice E (Lateral column of spinal cord white matter) is incorrect. All columns within the spinal cord white matter contain neuronal processes and sup- porting glial cells. Degeneration within the lateral column of the spinal cord would not include the somatic motor neurons involved in ALS.

Amyotrophic lateral sclerosis (ALS; Lou Gehrig's disease) is a progressive, fatal neurodegenerative disease caused by degen- eration of the motor neurons controlling skeletal (voluntary) muscle movement. Postmortem analysis of which of the fol- lowing structures would show the cell bodies of neurons affected by this disease? (A) Anterior gray horn of the spinal cord (B) Lateral gray horn of the spinal cord (C) Posterior gray horn of the spinal cord (D) Spinalganglia (E) Lateral column of spinal cord white matter

C: Long thoracic nerve. The photo demonstrates a case of "winged scapula," indicative of lesion of the long tho- racic nerve and subsequent paralysis of the serratus anterior muscle. The nerve is located on the lateral thoracic wall, on the superficial aspect of the serratus anterior, where it is not afforded the protection of the muscle it innervates (like most motor nerves), especially when the limb is elevated. In this patient, shrapnel wounds to the lateral thoracic wall caused damage to the long thoracic nerve and subsequent loss of innervation to the serratus anterior muscle. When the affected limb is protracted, the medial border and inferior angle of the scapula pull away from the posterior chest wall, giving the scapula a wing-like appearance. Additionally, the affected arm cannot be abducted above the horizontal plane because the serratus anterior is not available to superiorly rotate the gle- noid cavity of the scapula to allow full abduction. Following damage to the long thoracic nerve, it takes several weeks for a winged scapula to the develop because the trapezius muscle, which attaches to the spine of the scapula, must stretch before winging is apparent. Choice A (Axillary nerve) is incorrect. This nerve passes deeply through the axilla, around the surgi- cal neck of the humerus, to supply the teres minor and deltoid muscles. A lesion of this nerve results in significant weakness in abduction of the arm and wasting of the rounded contour of the shoulder, which is not reported in this patient. Choice B (Thoracodorsal nerve) is incorrect. This nerve runs downward through the axilla to supply the latissimus dorsi muscle. Loss of the nerve would result in weakness in extension and medial rotation of the arm, plus wasting of the posterior axillary fold. These signs and symptoms were not reported in this patient. Choice D (Dorsal scapular nerve) is incorrect. This nerve courses into the upper, medial part of the back and the lower neck to supply the levator scapulae and rhomboid muscles. Paralysis of these muscles would result in weakness in eleva- tion and retraction of the scapula, and perhaps wasting of the contour of the back under the trapezius muscle, which was not reported in this patient. Choice E (Suboccipital nerve) is incorrect. The suboccipital nerve is also the posterior (dorsal) ramus of the first cervical nerve (C1), which passes through the suboccipital triangle to innervate the rectus capitis poste- rior major and minor, obliquus capitis superior and inferior, rectus capitis lateralis, and semispinalis muscles. It is gener- ally believed that the suboccipital nerve contains only motor fibers, but it may receive afferent (sensory) fibers related to proprioception. Damage to the suboccipital nerve would result in weakness extending and rotating the head on the C1 and C2 vertebrae, which was not reported in this patient.

An 18-year-old soldier presents with shrapnel wounds in the lateral wall of his right chest following an explosion of a land- mine. After several months of recovery, his physical therapist observes that his scapula moves away from the thoracic wall when he leans on his right hand, as noted by the black arrow in the given photo. Which of the following nerves is likely damaged? (A) Axillarynerve (B) Thoracodorsalnerve (C) Long thoracic nerve (D) Dorsal scapular nerve (E) Suboccipitalnerve

B: Spinal ganglia. The posterior (dorsal) and anterior (ventral) roots of spinal nerves typically merge within the intervertebral foramina to form individual spinal nerves. Because the spinal (dorsal root) ganglia are located at the dis- tal ends of the posterior roots, they are also normally found within the intervertebral foramina. Thus, narrowing (steno- sis) or compression of the intervertebral foramina may cause impingement of the posterior and anterior roots, spinal nerves, and spinal ganglia due to the presence of bony spurs (osteo- phytes) in this patient. Choice A (Posterior primary rami of spi- nal nerves) is incorrect. Each spinal nerve divides into its first branches (primary rami) immediately after exiting the interver- tebral foramen. Because the primary rami are located distal to the intervertebral foramina, the posterior primary rami would not be compressed by stenosis of the intervertebral foramina due to the presence of osteophytes in this patient. Remember that the posterior primary rami extend into the epaxial tissues to innervate the deep (intrinsic; epaxial) muscles of the back. Choice C (Ganglia of the sympathetic trunk) is incorrect. The sympathetic trunk lies along the lateral side of the vertebral column, outside the intervertebral foramina. Thus, the gan- glia of the sympathetic trunk would not be impinged by the osteophytes found in this patient. Choice D (Anterior primary rami of spinal nerves) is incorrect. Each spinal nerve divides into its first branches (primary rami) immediately after exit- ing the intervertebral foramen. Because the primary rami are located distal to the intervertebral foramina, the anterior pri- mary rami would not be compressed by stenosis of the inter- vertebral foramina. Remember, the anterior (ventral) primary rami extend into the hypaxial tissues to innervate the superfi- cial and intermediate (extrinsic; hypaxial) muscles of the back. Choice E (Anterior rootlets of spinal nerves) is incorrect. The anterior (ventral) rootlets of spinal nerves are located proxi- mal to the intervertebral foramina, close to the spinal cord, and would not be impinged by a stenosis of the intervertebral foramina due to the presence of osteophytes.

An 81-year-old woman visits her family physician for an annual physical examination and complains of weakness and loss of sensation in her legs. Radiological studies show bony spurs (osteophytes) narrowing the intervertebral foramina at levels T12-L3. Which of the following structures is most likely to be impinged in this condition? (A) Posterior primary rami of spinal nerves (B) Spinalganglia (C) Ganglia of the sympathetic trunk (D) Anterior primary rami of spinal nerves (E) Anterior rootlets of spinal nerves

The answer is B: Midaxillary line—9th intercostal space. Secondary to her metastatic breast cancer, the patient is suffering from a pleural effusion (accumulation of excess fluid in the pleural cavity surrounding the lung). The effusion causes the difficulty in breathing due to limiting the expansion of the lungs during inspiration. A thoracentesis is an invasive procedure to remove the excess fluid from the pleural sac. In the midaxillary line, the lung resides at the 8th rib and the parietal pleura would reside at the 10th rib. The space between the parietal pleura and the lung is the costodiaphrag- matic recess, and it gives space for the lung to inflate within the pleural cavity. To drain the pleural effusion, a doctor must place the cannula (hollow needle) in a location where the lung and parietal pleura are separated. The 9th intercostal space at the midaxillary line is the ideal location to perform a thoracen- tesis because here the pleural effusion can be drained without damaging the lung itself. Choice A (Midaxillary line—6th intercostal space) is incorrect. Performing a thoracentesis at this location would likely damage the lung, which extends to the 8th rib in the midaxillary line. However, this location (6th intercostal space at the midaxillary line) is often used for placement of a chest tube, which is often inserted to remove air or fluids from the pleural space after the lung has collapsed due to a pneumothorax. Therefore, the lung tissue is no longer extending down to the 8th rib at the midaxillary line, so the drainage of the lung can occur at a higher level. Choice C (Parasternal line—6th intercostal space) is incorrect. Due to the majority of the heart residing on the left side of the tho- rax, the lung and parietal pleural are shifted away from the midline at this location. Therefore, a pleural effusion could not be drained at this location due to no access to the pleu- ral cavity. At this location, a pericardiocentesis could be per- formed to drain fluid from the fibrous pericardium. Choice D (Parasternal line—2nd intercostal space) is incorrect. At the 2nd intercostal space immediately to the left of the sternum, no space exists between the parietal pleura and the lung. Therefore, this location is not ideal to drain a pleural effusion. This location, however, is often used for auscultation of the pulmonary valve of the heart with a stethoscope. Choice E (Midclavicular line—5th intercostal space) is incorrect. At the fifth intercostal space at the midclavicular line on the left side, no space exists between the parietal pleura and the lung. Therefore, this location is not ideal to drain a pleural effusion. This location, however, is often used for auscultation of the mitral (or bicuspid) valve of the heart with a stethoscope

An 87-year-old woman with a history of metastatic adenocarcinoma of the breast presents with difficulty in breathing. A plain chest film reveals massive left pleural effu- sion, as noted in the photo. The physician chooses to drain the fluid with a thoracentesis. What is the proper location for placement of the cannula (hollow needle) to drain the pleural effusion? (A) Midaxillary line—6th intercostal space (B) Midaxillary line—9th intercostal space (C) Parasternal line—6th intercostal space (D) Parasternal line—2nd intercostal space (E) Midclavicular line—5th intercostal space

A: Ligamentum flavum. The ligamentum fla- vum (L: yellow ligament) consists of yellow elastic fibrous tissue, which binds together the laminae of adjoining verte- brae and forms the posterior wall of the vertebral canal. Dur- ing proper administration of an epidural anesthetic, the needle will pass (in order) through the supraspinous ligament overly- ing the spinous processes of the lumbar vertebrae, the inters- pinous ligament connecting the spinous processes of adjacent vertebrae, and finally the ligamentum flavum, which stretches between the laminae of adjacent vertebrae. Due to its high elastic fiber content and the usual placement of the pregnant woman into the fetal position, the anesthesiologist will feel substantial resistance before the needle passes through the ligamentum flavum and potentially an audible "pop" when it is penetrated. In the given diagram, the epidural space resides between the ligamentum flavum and the posterior longitudi- nal ligament. Remember that the ligamentum flavum is the last ligament the needle penetrates during administration of an epidural anesthetic. Choice B (Anterior longitudinal liga- ment) is incorrect. The anterior longitudinal ligament is a vertical connective tissue band that attaches along the ante- rior sides of the vertebral bodies, as noted in the given figure. During proper administration of an epidural anesthetic, the needle will pass (in order) through the supraspinous ligament, the interspinous ligament, and finally the ligamentum flavum. Due to its position residing anterior to the vertebral bodies, the anterior longitudinal ligament will not be penetrated by the needle during proper administration of an epidural anes- thetic. Choice C (Posterior longitudinal ligament) is incorrect. The posterior longitudinal ligament runs vertically along the posterior aspect of the vertebral column, as noted in the given figure. Due to its position residing posterior to the vertebral bodies, the posterior longitudinal ligament will not be pene- trated during proper administration of an epidural anesthetic. Choice D (Interspinous ligament) is incorrect. The inters- pinous ligament is composed of fibrous bands that connect the spinous processes of adjacent vertebrae, as noted in the given figure. This ligament is penetrated by the needle dur- ing proper administration of an epidural anesthetic; however, the ligamentum flavum is the last ligament penetrated by the needle to reach the epidural space. Choice E (Intertransverse ligament) is incorrect. The intertransverse ligament is one ligament that connects the transverse processes of adjacent vertebrae. Because the intertransverse ligaments do not lie in the median sagittal plane, these ligaments would not be penetrated by the needle during administration of an epidural anesthetic and cannot be appreciated on the given figure.

An anesthesiologist administers epidural anesthestic immedi- ately lateral to the spinous processes of vertebrae L3 and L4 of a pregnant woman in labor. During this procedure, what would be the last ligament perforated by the needle in order to access the epidural space? (A) Ligamentumflavum (B) Anterior longitudinal ligament (C) Posterior longitudinal ligament (D) Interspinousligament (E) Intertransverseligament

A: Notochord. The nucleus pulposus is the sole remnant of the embryonic notochord, the initial lon- gitudinal skeletal axis of the body. The developing bodies and intervertebral discs of the vertebral column replace the notochord, except for the nucleus pulposus. Remember that each intervertebral disc is composed of two parts: the cen- tral nucleus pulposus and the peripheral anulus fibrosus. Choice B (Dermatome) is incorrect. Each embryonic somite differentiates into three components: dermatome, myotome, and sclerotome. The dermatome contributes to the forma- tion of the dermis, most of which is derived from the somatic layer of the lateral plate mesoderm. The nucleus pulpo- sus component of an intervertebral disc is derived from the notochord, not the dermatome. Choice C (Myotome) is incorrect. Each embryonic somite differentiates into three components: myotome, dermatome, and sclerotome. The myotomes represent the muscle generating components of the somites. Remember, each myotome splits into two parts: epimere and hypomere. The epimere gives rise to the deep group of muscles in the back (intrinsic back muscles or epax- ial muscles), which develop dorsal to the incipient vertebral column and are innervated by the posterior (dorsal) primary rami. The hypomere gives rise to muscles located lateral and anterior to the vertebral axis, which are innervated by the anterior (ventral) primary rami of spinal nerves. The nucleus pulposus component of an intervertebral disc is derived from the notochord, not the myotome. Choice D (Sclerotome) is incorrect. The sclerotomes give rise to most of the vertebral column, including the vertebrae, part of the occipital bone, and the anulus fibrosus portion of the intervertebral discs. The nucleus pulposus component of an intervertebral disc is derived from the notochord, not the sclerotome. Choice E (Neural crest) is incorrect. Most of the vertebral column and immediately surrounding tissues are derived from mesoderm. However, neural crest cells do migrate into this area, where they form spinal (dorsal root) ganglia and autonomic ganglia. The nucleus pulposus component of an intervertebral disc is derived from the notochord, not the neural crest cells.

An infant suffers a vertebral malformation in which the nucleus pulposus component of multiple intervertebral discs is hyper- trophied. From what embryonic structure is the hypertrophied structure derived? (A) Notochord (B) Dermatome (C) Myotome (D) Sclerotome (E) Neuralcrest

E: S2. The medullary cone (conus medul- laris) is the tapered terminal end of the spinal cord, composed of the sacral and coccygeal segments. In adults, the conus medullaris typically lies within the T12-L3 vertebral levels, but it generally ends at approximately L2. Thus, displacement fractures of these vertebrae are likely to affect one or more of the sacral-coccygeal spinal cord segments. In this case, given the statistical range of variation for the medullary cone, the displaced fracture fragment of the L1 vertebral body is more likely to impinge the lower lumbar and upper sacral segments of the medullary cone. S2 is the only choice from this region. When thinking of the spinal cord, remember the "rule of 2," which states the conus medullaris usually ends at the second lumbar vertebra (L2); the subarachnoid space ends at the sec- ond sacral vertebra (S2); and the terminal filum (filum termi- nale), a long connective tissue (pia mater) strand extending from the end of the medullary cone that anchors the inferior aspect of the spinal cord, usually attaches to the second coc- cygeal vertebra (Co2). Due to the location of the displaced fracture fragment of the L2 vertebral body, the higher sacral segments of the spinal cord related to the medullary cone are more likely to be damaged. Choice A (T7) is incorrect. The normal range of variation for the location of the conus medullaris in adults is the T12-L3 vertebral levels. The sev- enth thoracic (T7) spinal cord segment lies posterior to the vertebral bodies of the T6 and T7 vertebrae. Therefore, the T7 spinal cord segment is located well above the displaced posterior fracture fragment of the L1 vertebral body and the medullary cone, and it would not be damaged in this patient. Choice B (T9) is incorrect. The ninth thoracic (T9) spinal cord segment lies posterior to the vertebral bodies of the T8 and T9 vertebrae. Therefore, the T9 spinal cord segment is located well above the displaced posterior fracture frag- ment of the L1 vertebral body and the medullary cone, and it would not be damaged in this patient. Choice C (T11) is incorrect. The eleventh thoracic (T11) spinal cord segment lies posterior to the bodies of the T10 and T11 vertebrae. Therefore, the T11 spinal cord segment is located well above the displaced posterior fracture fragment of the L1 vertebral body and the medullary cone, and it would not be damaged in this patient. Choice D (L2) is incorrect. The second lum- bar (L2) spinal cord segment lies posterior to the vertebral body of T12. Therefore, the L2 spinal cord segment is located well above the displaced posterior fracture fragment of the L1 vertebral body and the medullary cone, and it would not be damaged in this patient. The spinal nerve roots of L2 may be damaged by the displaced posterior fracture fragment; how- ever, this question asks specifically for the spinal cord segment that would be damaged.

Attempting to do a backflip with his bicycle off a high ramp at the finish line of a race, a 24-year-old professional BMX rider fell from a height of 20 ft and attempted to land on his feet. A sagittal CT reveals a burst fracture of the L1 vertebral body with a posterior displaced fracture fragment compressing the medullary cone (conus medullaris). Which of the following spinal cord segments would most likely be impinged by the bone fragment in this injury? (A) T7 (B) T9 (C) T11 (D) L2 (E) S2


Conjuntos de estudio relacionados

Real Estate Finance: Chapter 4 Noninstitutional Lenders

View Set

Chp 19: Documenting and Reporting (PrepU)

View Set

International bissiness quiz 2, ch 3-5

View Set

RICA subtest 1 - RICA practice Questions

View Set

AP Computer Science Principles Semester 1 Exam Review

View Set

Chapter 18 Terms, Peoples, and Events

View Set